‘To heat a planetary surface’ for dummies; Part 4

I rounded off Part 3 of this series by suggesting the following:

Next up: How do you heat a planetary surface, then? If not by the Earth’s own thermal radiation, a result of its temperature rather than a cause of it … How does the atmosphere insulate the surface?”

Not so. This will have to wait a bit still. Next post, perhaps. I will rather try to clarify my stance on the whole ‘bidirectional flow’ concept thing, seeing how this topic has a tendency of stirring up both emotions and misconceptions.



There is quite a bit of confusion surrounding the whole issue of electromagnetic radiation, the Stefan-Boltzmann Law and the thermodynamic concept of ‘energy transfer’.

I will try to explain why there can be no such thing as a bidirectional energy transfer between two objects radiating at each other. Yes, they are radiating at each other! Radiation goes in all directions.

But, radiation in itself does not universally constitute a thermodynamic ‘energy transfer’. For instance, a cool object cannot and will not transfer any of its energy via radiation to a warmer object. This should go without saying, since this would be a direct violation of the 2nd Law of Thermodynamics.

Recall that a thermodynamic ‘energy transfer’ is always unidirectional, always comes in the form of either ‘heat’ [Q] or ‘work’ [W], and always changes the ‘internal energy’ [U] (and therefore, normally, the temperature) of the two objects/regions involved in the transfer.

The confusion simply arises from not being able to distinguish between a potential ‘energy transfer’ and an actual (real) ‘energy transfer’.

A cool object would always potentially be able to transfer energy to any other object, simply from the basic fact that it radiates energy in all directions by virtue of it having a temperature above absolute zero.

However, if a nearby object happens to be warmer than our first one, then this potential will not be realised in that particular direction. Yup, it still radiates in the direction of the warmer object, but it does not and cannot transfer energy to it. In this case, its radiation is a potential ‘energy transfer’ only.

Sounds like a contradiction in terms? Counter-intuitive? Yes. I agree. But this subject is a strange one. It is quantum theory stuff. Things we cannot really see, only imagine.

The reason why radiation doesn’t at all times necessarily translate into an energy transfer, however, is not that hard to explain.

What you have to realise is that the ‘bidirectional flow’ concept of radiative heat transfer is all about mental images. It is a mental model constructed solely for the purpose of trying to grasp with our human mind what is actually going on.

So we mentally and hypothetically split up an observed physical process into separate, individual parts. As if these parts or pieces were independent from one another, somehow working all by themselves within the process examined, the process in our case of course being a radiative heat transfer. We look first at the one piece, then at the other, and then finally we put the theorised individual effects of these two pieces into one ‘net effect’. Working backwards, so to say. After all, we already know the ‘net effect’. It is what we actually observe and detect in the real world. We simply imagine this to be the result (the ‘sum’) of two opposing constituent effects.

And in a way, this is true. But in saying so, one should not forget that the two individual effects making up the ‘net’ aren’t actually real (in the sense of being realised) effects. They are merely potential effects. The only real (realised) effect is the one we call the ‘net’, the one we actually experience.

It is absolutely essential to understand the distinction here: Real/actual (observed) vs. potential (hypothetical) effects.

In reality, the potential effect of our two imagined individual processes would only become real if the opposite process didn’t occur at the same time, occupying the same space.

Relating this to the Stefan-Boltzmann Law, it is like pointing out that the cool object would have transferred its energy by radiation, based solely on its own temperature and emissivity, to the warm object if (and only if) the warm object were rather an object (or surroundings) at absolute zero. So that it didn’t radiate anything back. This is in fact precisely what the radiative heat transfer equation is saying, in going from this form:

q = σT4

to this:

q = σ(Th4 – Tc4)

We realise that (in the latter version) the q term on the lefthand side is the actual (observed, detected) ‘energy transfer’, while the T terms on the righthand side are just potential (hypothetical) energy transfers. They are not individually realised any of them. A T term on the righthand side is only ever realised as an actual energy transfer if it stands alone, like in the upper equation.

So we call the actual ‘energy transfer’ (the heat transfer), the q, a ‘net result’. But is it really? Well, yes, it is hypothetically and mathematically the net of two opposing potentials. It is, however, not the net of two real, separately working physical processes. There is only the one process realised – the heat transfer.


This might still not make much sense. The cooler object is after all radiating energy in the direction of the warmer one. So how could it not transfer any energy to it?

First of all, because it simply doesn’t happen in nature. It is prohibited by the fundamental laws of thermodynamics. You cannot have a situation where energy from a cooler object is transferred to a warmer one and have that energy, directly and all by itself, increase the ‘internal energy’ and thus the temperature of the warmer one in absolute terms. A situation like with the ‘Steel Greenhouse’, or with the postulated rGHE for Earth. As explained here.

This would be plainly at odds with the 2nd Law. There is no way around it.

So we have to find another way of explaining what happens in a radiative heat transfer. There simply is no and cannot be any ‘bidirectional transfer of energy’ between two radiating objects, because a ‘transfer of energy’ in thermodynamics has a very specific meaning; it is a very specific process yielding a very specific effect. One which can only go one way, in a heat transfer situation, from hot to cold.

Secondly, it is because of the fundamental nature of light. Light moves like a propagating wave, a set of wavefronts at a particular wavelength and amplitude, through space. Energy transfers with the waves. So if the wavetrain moves in one direction, the energy moves in that same direction.

When two waves moving in the opposite direction meet, however, they interfere. If they have the same wavelength and amplitude (containing the same amount of energy), they create a so-called standing wave between them. The standing wave is the ‘net result’ of the two opposing waves:

1d_wave_equation_animation

Animation 1.

The oppositely moving green and red waves portrayed here would not be observed in the real world. They basically do not exist as physical entities. They exist separately and independently only up to the point where they meet. After this point, their individual physical existence is lost. Their potentials carry on, though. But only the ‘net result’ of the two, the blue standing wave pattern, is to be observed. Only that is realised.

This is not a trivial point. It is THE point to make.

Once the standing wave pattern is up, there is no further transfer of energy between the two sources. Because the standing waves between them are not moving in either direction. The energy was travelling freely through space until the two opposing wavetrains met. Then it stopped. You can observe what happens here:

Two_sources_interference

Animation 2.

The two point sources appear to be of equal strength. Watch what happens to the waves they send out toward each other. They meet halfway and then there is no further horizontal movement. No energy from the one source ever reaches the other.

In a heat transfer situation, this would represent a thermal equilibrium. The two objects have reached the same temperature and so no more energy (heat) can be transferred between them.

You would have the same thing with perfect reflection. The reflected energy from the source does not move back to the source. It is ‘caught’ in a standing wave in the space between. No energy transfer.


So what happens in a radiative heat transfer where a warm object heats a cooler one, like say from an initial temperature of 0 K to a final temperature equal to that of the warm object?

We can opt to explain this in two ways:

  1. A wavelength-specific amplitude surplus resulting in a partial standing wave.
  2. A wavelength-specific ‘photon’ surplus resulting in a certain ratio between standing waves cancelling and travelling waves transferring.

Now, bear in mind that both of these explanations/descriptions are most decidedly merely conceptual models – simplifications of reality – just as much as the ‘bidirectional flow’ concept of radiative heat transfer. The difference is, these two have the advantage of not ending up violating the 2nd Law of Thermodynamics 😉

1. An amplitude surplus (‘The block approach’)

If two facing planar blackbody surfaces at different temperatures radiate at each other, we can represent each specific wavelength band of their respective EM spectrums as one ‘total’ wave. The energy transferred by this ‘total’ wave is determined not by its wavelength (which, after all, is already set by its spectral band), but rather by its amplitude. This would make it equivalent to an ocean wave or a sound wave, but not really to an individual electromagnetic wave, whose energy is derived specifically from its wavelength: E = hc/λ.

The one ‘total’ wave is thus basically rather an abstract representation of the sum of all the individual EM waves emitted in the given spectral band, and therefore of the total energy transferred from that band, for the blackbody in question, determined only by its temperature.

Since a warmer blackbody would emit more energy at every wavelength than what a cooler blackbody would, we can know that the amplitude of the ‘total’ wave at each particular wavelength emitted by it would be larger than the amplitude of the equivalent ‘total’ wave emitted by the cooler blackbody.

We would then end up with a set of ‘partial standing waves’ moving unidirectionally from the warmer BB to the cooler BB.* The result of which would be the actual ‘energy transfer’, the radiative heat.

*A full standing wave corresponds to the case where the amplitudes of the forward and backward-travelling waves (of equal wavelength) are equal. Conversely, a regular travelling wave corresponds to the case where one amplitude is zero.

We therefore end up going from a set of full, regular travelling waves from the warmer object to the cooler one at t(T>> T(0 K)) to a set of full, regular standing waves between the two at tf (Tw = Tc). From full transfer to no transfer of energy.

2.  A ‘photon’ surplus (‘The swarm approach’)

This explanation/description is basically the same as the first one, only via a different route. Instead of adding up all the individual EM waves emitted in a given spectral (wavelength) band into one ‘total’ wave of a certain cumulative amplitude before letting it face the opposing wave, one could rather ‘count’ the total number of individual waves (or ‘photons’) emitted in that spectral band and stack each of them up against the individual waves/’photons’ emitted by the opposing object of the same band and see who comes out on top.

Since all individual EM waves within a particular wavelength band naturally have the same wavelength and hence carry the same amount of energy, there will be a multitude of standing waves between the two opposing ‘wave streams’. The warmer object of the two, however, will emit a higher total number of ‘photons’ (a higher energy density) toward the cooler than the cooler emits toward the warmer, so even as all the ‘photons’ from the cooler object is tied up in standing waves with equal energy ‘photons’ from the warmer object, the warmer object will still have ‘photons’ to spare. And these ‘photons’, then, will meet no opposition at all. They will transfer their energy like regular travelling waves to the cooler object.

This ‘photon’ surplus (like the amplitude surplus of the ‘block approach’ above) will become smaller and smaller the closer the temperatures of the two objects get, in the end reaching nil.

The end result of this conceptual model description will be exactly the same as in the first one – because they both simply describe (and attempt to explain) the actual (observable) ‘energy transfer’, the radiative heat.


This superposition of waves works at all points in time and space throughout the integrated radiation field between the two planar BBs, from the one surface to the other. When the ‘net’ wave pattern is set up (happens spontaneously and instantaneously as soon as the two objects are brought into thermal contact), then its movement through the field is what ‘pulls’ the energy from the warmer object and ‘deposits’ it at the cooler.


‘Photons’ versus EM waves

At this point it feels as if a brief discussion on ‘photons’ is warranted.

What is a ‘photon’? Is it something other than an EM wave? Well, its energy tends to be defined in a different (albeit mathematically interchangeable) way from that of a single EM wave, by its frequency: E = hν; an EM wave’s energy would rather normally be defined by its wavelength: E = hc/λ. It would make little sense defining the energy of a single EM wave by frequency. Why? Well, can a single wave be said to possess a frequency? What is ‘frequency’ in the first place? It is the passage of a certain number of waves through a specified interval over a particular period of time. So you can basically only measure the frequency of light from a fixed point outside the actual light beam. If you travel with an individual EM wave flying forward at the speed of light (duh!), then you could only ever assess its length; the wave itself would not come with a frequency. Mathematically, we can of course determine a hypothetical frequency as the speed of light (c) divided by its wavelength (λ), but it doesn’t really mean anything physical. Not for that single wave. It just means that, since all EM waves travel at the speed of light (they are light, after all), we can know for sure that it is their wavelength alone that will determine in the end how many of them will pass through our designated interval. Hence the simple relation: c/λν.

So you see the artificial (and ultimately unnecessary) distinction being made this way between an EM wave and a ‘photon’ …

Defining the energy of the ‘photon’ by its frequency seems to set it apart as a separate entity from the EM wave. As if light is the combined product of two different things put together, a wave and a particle. In most people’s minds, I’m sure, a ‘photon’ is a particle as good as any other elementary particle, a concrete ball of substance flying independently around. In this ‘wave-particle duality’ perspective on the nature of light, one gets the distinct feeling that the ‘photon’, the particle, is meant to sort of be riding on the back of the waves; not on the back of a single wave, but along the rollercoaster course set up by the wavetrain in its entirety. One gets the sense that the wavetrain is somehow frozen in time and space, functioning only as the cresty and troughy highway along which the ‘photons’ fly. In this perspective, the ‘photon’ is the sole carrier of light’s energy, while the waves are just there to provide it with a means to transfer this energy from one place to the next.

This clearly seems to be the corollary of the “‘photon’ gets its energy from its frequency” approach. I’m sure the old masters never meant for it to be this way. But it is. The energy of the ‘photon’ is ostensibly determined by how many individual (and – one must assume, then – unmoving) waves it can pass within a certain amount of time, usually a second. Since presumably it always travels at the speed of light, the length of the waves it passes underway, then, determines its energy.

This all comes off as weird. For instance, if a ‘photon’ travels along the path of an undulating wavetrain, how could it ever manage to move forward in space at the speed of light (c)? Wouldn’t it then have to move considerably faster than the speed of light, along its actual path, in order to cover the distance from wave crest to wave crest at the speed of light:

purewave

Figure 1.

Such a view is of course absurd.

No, the ‘photon’ is simply an individual light wave, one distinct EM wavefront. It is the wavefront itself that moves forward at the speed of light, not something moving along its length.

The wave and the particle are one and the same. The ‘photon’ simply represents a particular property of EM waves, namely how they come in discrete packets rather than in a continuous stream like mechanical (macroscopic) waves do.

Light is a strange thing. No point in making it even stranger …

What this entails is the following: Rather than being ‘unstoppable’ particles of light, travelling forward no matter what happens, no matter what the waves might do, ‘photons’ simply represent the quanta of light energy moving with and in each single EM wave in the train. So if the waves stop moving forward, like in a standing wave pattern, then the ‘photons’ (and thus the energy) naturally stop moving forward also. The ‘photon’ is not a separate thing from the EM wave.

This situation is in fact summed up quite well by Wikipedia:

“Eventually the modern theory of quantum mechanics came to picture light as (in some sense) both a particle and a wave, and (in another sense), as a phenomenon which is neither a particle nor a wave (which actually are macroscopic phenomena, such as baseballs or ocean waves). Instead, modern physics sees light as something that can be described sometimes with mathematics appropriate to one type of macroscopic metaphor (particles), and sometimes another macroscopic metaphor (water waves), but is actually something that cannot be fully imagined.”

Yes, it’s both and neither, in one.



So, finally, if we had two opposing blackbody spectrums (Figure 2 below), what would be the ‘net’ result of them coming together? Quite simply the ‘energy transfer’ actually observed between the objects emitting them (Figure 3 below). The ‘hot’ spectrum minus the ‘cold’ spectrum. At each wavelength. The result: the radiative heat from hot to cold. There is no double transfer anywhere. Only two potential transfers resulting in one real (as in ‘realised’) transfer: q, the heat.

Everything but the heat is purely conceptual. It doesn’t really exist, doesn’t really occur. Only on paper. And in our minds. Nowhere in the real world.

two-planck-curves

Figure 2.

difference-of-two-planck-curves

Figure 3. Both of these figures, I will freely admit, are borrowed (for their clarity) from Joe Postma’s blog.

The black curve of the resulting actual ‘energy transfer’ – the radiative heat, q – between the hot and the cold object, does not in itself represent a blackbody emission spectrum like the other two (the potentials), even though it mathematically emulates the shape of one. The energy moving between two warm objects as heat, after all, is not emitted as radiation by any one of the two objects (nor by a third and unknown ‘heat emitter’). That doesn’t make any difference. The heat is still the only real flux (transfer) of energy around.

How can you tell, then, that the black curve in Figure 3 does not itself represent a third BB spectrum? Apart from the fact that two spectrums could never magically generate a third one between them. Because how would that work …?

You can tell it straight from the graph. The peak wavelength of the ‘heat curve’ is shifted further to the left than the peak of the ‘hot’ spectrum. Which makes no physical sense. Mathematically, of course, this is just the result of subtracting the T1 curve from the T2 curve. But physically it has no meaning. As if the heat flux from the hot to the cold object were emitted by an object even hotter than the hot one, only putting out much less total energy (the area below the curve), as if it were somehow hotter, but much downscaled (like the solar flux at 1AU).

72 comments on “‘To heat a planetary surface’ for dummies; Part 4

  1. Kristian,

    For all classical thermodynamics or even post normal science, your concepts of energy, mass, and temperature are quite consistent. Why do you think that the most relativistic electromagnetic power transfer between locations has anything to do with your concepts!

    • okulaer says:

      May I ask, Will, what is this “most relativistic electromagnetic power transfer between locations” of yours? I presume it’s the Q/q (or P, P/A) you’re talking about. Is this quantity something we can detect? And if so, how do we detect it? What does it do? What effect does it have on those different ‘locations’? Anything to do with temperature?

      Finally, how does it relate to the (thermodynamic) Q in the 1st Law?

      To me it looks like your only real objection is to my calling it ‘heat’. You want to call it something else. But we’re talking about the exact same phenomenon. Being the same. Doing the same. It is what it does that matters, Will. Not what we prefer to call it.

      • “May I ask, Will, what is this “most relativistic electromagnetic power transfer between locations” of yours? I presume it’s the Q/q (or P, P/A) you’re talking about. Is this quantity something we can detect?”

        Easy to detect, electromagnetic power (even undefined)! The conversions of thermal EMR to charging a battery, accumulation of useful power or delivery of power as angular momentum, latent heat of evaporation, tree conversion of EMR to vast hydrocarbon structure. Your temperature dependent “heat” (Q) cannot do that except in the trivial case. Your damned (Q) is but entropy times temperature for any and all mass.

        ” And if so, how do we detect it? What does it do? What effect does it have on those different ‘locations’? Anything to do with temperature?”

        See the above!

        “Finally, how does it relate to the (thermodynamic) Q in the 1st Law?”

        Your concept of conservation of energy (Q) is well demolished by E. Noether in 1917.

        “To me it looks like your only real objection is to my calling it ‘heat’. You want to call it something else. But we’re talking about the exact same phenomenon. Being the same. Doing the same. It is what it does that matters, Will. Not what we prefer to call it.”

        Your what you “call it” results in “back radiation” with no attempt whatsoever at understanding!

        Your accumulation of power, mass velocity, location in a gravitational field, the temperature of mass with specific heat, is the FRAUD! This physical “is” must be different perhaps not even understandable. Time to distinguish “Earthlings that claim to know”, from all the rest that are only peddling as fast as they can.

  2. claesjohnson says:

    Hi Kristian

    You can find ideas similar to what you present on

    https://computationalblackbody.wordpress.com/

    Best regards Claes
    Johnson

  3. “Yes, they are radiating at each other! Radiation goes in all directions.”

    What nonsense. Thermal “radiance” is a field strength, a potential, for emitting or absorbing EMR in each direction. It is never your claimed “goes” in all directions. The field is, only, a field and does not ever go. A field is developed in space, with no need for any power maintenance. What may go “the actual power transfer per unit area (flux)” can only be a result of and proportional to the difference in such opposing potentials. Nothing else has ever been detected, observed, nor measured. Radiation (flux) need not “go” in all directions.

    • okulaer says:

      Read the whole post, Will. What you say is exactly what the post says. You just need to keep the concepts apart.

      • “Read the whole post, Will. What you say is exactly what the post says. You just need to keep the concepts apart.”

        Not at all. You have concepts. I have measurements. You “Say” with nothing behind your statings.

        You insist that your POV is correct. I sit here scratching head or more intimate body parts. I do not know. However I look, I measure, I ponder, then I get dronk!

      • Kristian,
        You complain of the blind spots of others, but cannot comprehend your own. Neither the sphere nor the shell need any mass or specific heat thus no Q and no U,. The work of EMR is power applied over “any” distance as space is reactive not dispersive. This atmosphere is much much different.
        I have again read the whole post. It is scientific nonsense just like Postma. You take all that has been demonstrated of EMR and turn it on its head in order to claim some symbolic algebra is the first law of thermodynamics. An algebra can assist in understanding a law but only with a rigid understanding of each symbol. You cannot give a coherent distinction to your Q,W,and U.
        U = (Q-W), W = (Q-U), Q = (U+W)? Give me a break. Energy is never the result of Work plus entropy, or Work plus “internal energy”. This physical never does that! Your symbolism only works for the symbols, thus cannot be any assistance to understanding a law. Force times distance cannot be equated to energy in this physical, with its required losses to entropy. Your invention of internal energy cannot be energy at all while contained as entropy at the lowest local temperature.

        “Yes, they are radiating at each other! Radiation goes in all directions.”

        The same claim over and over again, with no definition of “radiation”, just like a warmist claim. A radiative field strength can be omnidirectional, with an isotropic radiator. Field strength from one radiator in the presence of others at the same frequency, but without coherence, never determines flux. Flux can never be greater that the vector sum of all field strength vectors at any frequency and location. Any “standing wave” indicates only a coherent reflection of some field strength, never some flux.

        • okulaer says:

          Sorry, Will, but it’s pretty hard to take part in any meaningful discourse on any thermodynamic subject with someone who doesn’t at all appear to accept regular, standard concepts, principles, definitions and terminology of the field of … Thermodynamics. Who seemingly hasn’t opened a textbook on Thermodynamics in 60 years because he’s decided it’s all part of some grand-scale hoax to dupe the free people of the world. When absolutely nothing has changed since the time of Clausius and Gibbs and Helmholtz and Maxwell and Thomson and Stefan and Boltzmann and Planck, except how the different terms designate the different phenomena described. ‘Heat’ is transferred from one body to another (at a lower temperature) also by way of radiation, Will. You need to live with this. Even J.C. Maxwell points this out:

          “(…) we (…) direct our attention to the process by which heat is transferred from one body to another.

          This process is called the Diffusion of Heat. The diffusion of heat invariably transfers heat from a hotter body to a colder one, so as to cool the hotter body and warm the colder body. (…)

          (…) We are at present concerned only with the passage of heat into the body or out of it, and this always takes place by diffusion, and is always from a hotter to a colder body.

          Three processes of diffusion of heat are commonly recognised – Conduction, Convection, and Radiation.”

          My “radiative heat” (Maxwell’s “radiant heat”), Stefan-Boltzmann’s “Q”, and your “EMR power”, is ultimately the exact same thing. We just call it by different names. There is no physical difference. Names. Terminology. Semantics.

          You say, Will: “Flux can never be greater that the vector sum of all field strength vectors at any frequency and location. Any “standing wave” indicates only a coherent reflection of some field strength, never some flux.”

          Precisely. There is no radiative “flux” moving from the cooler to the warmer object. As my post explains. For a ‘flux’ is a ‘transfer of energy’. And in a heat transfer, such a transfer ALWAYS and ONLY occurs from hot to cold, when it comes to EM radiation, the ‘net’ of the two opposing radiative ‘potentials’ (or ‘field strengths’, if you will).

          I’m afraid you’re tilting at windmills, Will. You’re reading my posts with your blinkers on. Try to look for solutions rather than problems (scamsters round every corner). You need to find someone else than me to be your enemy …

      • okulaer says:February 20, 2015 at 11:29 pm

        “Sorry, Will, but it’s pretty hard to take part in any meaningful discourse on any thermodynamic subject with someone who doesn’t at all appear to accept regular, standard concepts, principles, definitions and terminology of the field of … Thermodynamics. Who seemingly hasn’t opened a textbook on Thermodynamics in 60 years because he’s decided it’s all part of some grand-scale hoax to dupe the free people of the world.”

        Kristian,
        I read modern textbooks on heat transfer, every day and am appalled at the lack of any personal integrity, or due diligence, on the part on the writers and reviewers. It is no conspiracy,
        It is but Grand Sloth on the part of modern academia. Please read my now two posts to Mark Stoval trying to get him to understand your POV, without destroying his good POV.

        “When absolutely nothing has changed since the time of Clausius and Gibbs and Helmholtz and Maxwell and Thomson and Stefan and Boltzmann and Planck, except how the different terms designate the different phenomena described.”

        Indeed the electrical has sought to differentiate and identify. We now have lasers that have no dependency on thermal mass or thermometric temperature. There is no thermodynamic principal that can describe this transfer of this electromagnetic power in a direction of higher thermometric (potential) temperature. Your mechanical thermodynamics has since 1960 only sought to combine concepts, loosing details, and eventually dumbing down all, including Physics Lecturers.
        You Kristian are much better than that, by your postings, you identify the differences, but insist on poking all into the mechanical thermodynamic hamper.

        ‘Heat’ is transferred from one body to another (at a lower temperature) also by way of radiation, Will. You need to live with this. Even J.C. Maxwell points this out: “(…) we (…) direct our attention to the process by which heat is transferred from one body to another.
        This process is called the Diffusion of Heat. The diffusion of heat invariably transfers heat from a hotter body to a colder one, so as to cool the hotter body and warm the colder body. (…)
        (…) We are at present concerned only with the passage of heat into the body or out of it, and this always takes place by diffusion, and is always from a hotter to a colder body.
        Three processes of diffusion of heat are commonly recognized – Conduction, Convection, and Radiation.”

        Yes indeed, then the mechanics of “heat engine” were well known, a steam powered water pump was patented 100 years before. Maxwell was trying for understanding in terms of those uncomfortable with the terms EMR, caloric, phlogiston:
        Conduction is spontaneous diffusion in a direction of lower linear thermometric potential.
        Convection is conduction plus, mass transfer, powered by anything including electricity.
        Radiation, (thermal) after 140 years is one of the few power transports that depends not at all on any of the laws of thermodynamics, (0..3), but only on its own potential differences, including the concept of emissivity, reflectivity, and transmissivity, which “must” always sum to unity. Please demonstrate how this is similar to diffusion?.

        “My “radiative heat” (Maxwell’s “radiant heat”), Stefan-Boltzmann’s “Q”, and your “EMR power”, is ultimately the exact same thing. We just call it by different names. There is no physical difference. Names. Terminology. Semantics.”

        Ah! an admission of your own blind spots!

        ‘You say, Will: (radiative)” (“Flux can never be greater that the vector sum of all field strength vectors at any frequency and location. Any “standing wave” indicates only a coherent reflection of some field strength, never some flux.”)

        “This process is called the Diffusion of Heat. The diffusion of heat invariably transfers heat from a hotter body to a colder one, so as to cool the hotter body and warm the colder body. (…)”

        Not at all, Thermal electromagnetic radiative flux, is only concerned with the spontaneous discard of excess sensible heat or entropy in any direction of lower radiance at any frequency.
        Thermal electromagnetic radiative flux does not give a shit about what the other end may do with such flux, Such flux (loss of entropy) may never be absorbed by anything. So what?
        Please read the works of Emilly!

        “(…) We are at present concerned only with the passage of heat into the body or out of it, and this always takes place by diffusion, and is always from a hotter to a colder body.”

        Ok correct for, a thermodnamicist! The conjugate of Climate Clown! (negation and inversion).

        Three processes of diffusion of heat are commonly recognized – Conduction, Convection, and Radiation.”

        Ok mostly correct, the others measured are capacitive and inductive heat transfer, always limited to small distance with no mass, or maybe not, with what the Sun is doing to the Earth now.

        “My “radiative heat” (Maxwell’s “radiant heat”), Stefan-Boltzmann’s “Q”, and your “EMR power”, is ultimately the exact same thing. We just call it by different names. There is no physical difference. Names. Terminology. Semantics.”

        Each can be positively identified, and differentiated from all others, easily.

        “You say, Will: “(radiative) Flux can never be greater that the vector sum of all field strength vectors at any frequency and location. Any “standing wave” indicates only a coherent reflection of some field strength, never some flux.””

        I can demonstrate that!

        “Precisely. There is no radiative “flux” moving from the cooler to the warmer object. As my post explains. For a ‘flux’ is a ‘transfer of energy’. And in a heat transfer, such a transfer ALWAYS and ONLY occurs from hot to cold, when it comes to EM radiation, the ‘net’ of the two opposing radiative ‘potentials’ (or ‘field strengths’).”

        Indeed, but this has absolutely nothing to do with thermodynamics, only Maxwell’s equations.

        “I’m afraid you’re tilting at windmills, Will. You’re reading my posts with your blinkers on. Try to look for solutions rather than problems (scamsters round every corner). You need to find someone else than me to be your enemy …
        I wish no more enemies, I wish you all that you consider nice! 🙂 You seem to be one of the very few that can appreciate an alternate POV.

      • okulaer says:

        The point is, Will, our fundamental POVs on this issue don’t differ. We agree. You simply do not accept my (and the entire field of Thermodynamics’) use of the term ‘heat’ and I (and the entire field of Thermodynamics) do not adhere to your archaic and outdated (since a hundred years and counting) use of the term ‘heat’. (Note, the PHENOMENON hasn’t changed. We simply use a new WORD for it.)

        The 1st Law of Thermodynamics expressed mathematically for a closed system: ΔU = Q – W.

        If you cannot even grasp what this simple law/equation is saying (and has been saying since the time of Clausius himself) and what its different terms signify, how it relates, for instance, directly to the adiabatic process, then I cannot help you, Will. Then we can never be on the same wavelength.

        So I will not argue this point with you anymore. Because it’s proven completely fruitless, only counterproductive.

        Enough.

  4. Mr Pettersen says:

    This is easely demonstrated with a glas of water. If you poor it in the snow the water will give of energy to the snow. If you poor it in boiling water it will cool the boiling water. The water in the glas will always have the same amount of energy but nothing happens if you mix it with water with same temperature.

    So what the energy in the glas of water will do depends on the energy level it meets. Thats why its not possible to just add the energy from one source to another.

  5. markstoval says:

    “… But, radiation in itself does not universally constitute a thermodynamic ‘energy transfer’. For instance, a cool object cannot and will not transfer any of its energy via radiation to a warmer object. This should go without saying, since this would be a direct violation of the 2nd Law of Thermodynamics. … Recall that a thermodynamic ‘energy transfer’ is always unidirectional, always comes in the form of either ‘heat’ [Q] or ‘work’ [W], and always changes the ‘internal energy’ [U] (and therefore, normally, the temperature) of the two objects/regions involved in the transfer.” ~ okulaer

    I can agree with this line of thought. It is what I was trying to say in the thread on an earlier post. I see your words as saying that given two different masses in isolation in deep space that the warmer one would transfer heat to the cooler one as long as the first was still “hotter” than the second.

    And so, in the steel “greenhouse” the sphere transfers heat to the shell and the shell transfers heat to deep space. There is no heating of the sphere. There is no “slowing down” of the radiation from sphere to shell as the shell is radiating to deep space just as fast as the original sphere was before the shell got there.

    Anyway, this is how I see it. I am not any kind of expert on this at all.

    • okulaer says:

      Yes, I realise I can’t get past your (and Postma’s) blind spot concerning this problem.

      The cooler shell does not send back any energy to the warmer sphere to make it even warmer. On this we agree. However, the sphere must get warmer. By simple energy budgeting. By simple arithmetic.

      It is the warmer sphere’s own heat source (its internal power source), not its insulating layer (the shell), that provides the energy to make it both warmer and warmer still. Because it supplies it with a constant heat input [Q_in] from within, while the presence of the cooler shell (albeit, importantly, warmer than space, the sphere’s previous surroundings), by the Stefan-Boltzmann radiative heat transfer equation (Q/A or q = σ[Th^4 – Tc^4]), reduces the sphere’s heat output [Q_out] to its outside surroundings. With the sphere’s Q_in > Q_out, the result is an increase in the sphere’s U and hence its T. This is such a simple principle, Mark. The principle of insulation.

      But apparently I can keep telling you this over and over and over again and it just bounces off, simply ignored. It’s not that it doesn’t sink in. It is simply disregarded. This is your specific blind spot, Mark. You haven’t once addressed the physical relation I’m pointing to in the above paragraph. Because you simply see past it every single time. Lalalalala. It’s like the argument was never made at all. It is, however, what Part 3 of this series was all about. If you happen to remember …

      Well, I can’t tell you more than this, Mark. I’m sorry. If you still can’t see it, then I’m afraid I can’t help you.

      • markstoval says:

        “Well, I can’t tell you more than this, Mark. I’m sorry. If you still can’t see it, then I’m afraid I can’t help you”

        That is ok. I am still enjoying the series and I still keep an open mind that I may be wrong. I am looking forward to your post on how the planet is really warmed. I hope it is coming soon.

    • “And so, in the steel “greenhouse” the sphere transfers heat to the shell and the shell transfers heat to deep space. There is no heating of the sphere. There is no “slowing down” of the radiation from sphere to shell as the shell is radiating to deep space just as fast as the original sphere was before the shell got there. ”

      Change heat, radiation, and radiating to electromagnetic flux, and you and you are correct.
      EMR is never ‘a thermodynamic ‘energy transfer’, it is but an electromagnetic power transfer.
      Say the area of the shell were twice the area of the sphere. the fixed power applied to the sphere to space would result in a temperature (Tsp) higher by the fourth root of 2 ‘(1.19)’, than the temperature of the same fixed power applied to the shell to space (Tsh), this is entirely do to the change in power/surface area.
      Nest the sphere inside the shell and apply power to the sphere. The resulting temperature of the shell (Tsh) is exactly the same because of the same power same area and same opposing radiance (almost zero). The temperature of the sphere (Tsp)’ however, must go to (Tsh) times the forth root of (2+1) ‘(1.316)’. Even if the shell had only 1% more area with vacuum between,
      the sphere temperature must go to the forth root of (1.01+1) not the forth root of (1.01).
      The difference is that the shell must convert the EM flux back to sensible heat and temperature before re-radiating outward. Thermal EMR is not linear with temperature nor is it heat. EMR has its own laws. The fact that these laws also hold for the laws of thermodynamics when heat and temperature is involved was nice, and the subject of most communication between M. Planck and J. Poynting. Note your microwave oven can boil water but no part has a thermometric temperature as high as 100 Celsius
      Kristian complains of the blind spots of others, but cannot comprehend his own. Neither the sphere nor the shell need any mass or specific heat thus no Q and no U,. The work of EMR is power applied over “any” distance as space is reactive not dispersive. This atmosphere is much much different.

      • markstoval says:

        “And so, in the steel “greenhouse” the sphere transfers heat to the shell and the shell transfers heat to deep space. There is no heating of the sphere. There is no “slowing down” of the radiation from sphere to shell as the shell is radiating to deep space just as fast as the original sphere was before the shell got there. ”

        Change heat, radiation, and radiating to electromagnetic flux, and you and you are correct.

        Then you agree with this? —> in the steel “greenhouse” the sphere transfers electromagnetic flux to the shell and the shell transfers electromagnetic flux to deep space. There is no heating up of the sphere. There is no “slowing down” of the electromagnetic flux from sphere to shell as the shell is giving off electromagnetic flux to deep space just as fast as the original sphere was before the shell got there.

        Would that paragraph be correct in you opinion?

      • “Would that paragraph be correct in you opinion?”

        Yes quite precise from a electromagnetic POV that ignores thermal mass for the sake of clarity.
        From a thermodynamic POV, the mass and specific heat of the shell cannot be ignored, which always causes some delay in reaching equilibrium in both the shell and sphere. During this delay, however, a much different approach is needed like a capacitor connected with a resistor to another capacitor with a resistor to a constant voltage, all initial conditions must be evaluated.

        Mark,
        Please take the time to evaluate the equilibrium temperature of both the sphere and the shell as the shell grows in surface area and approaches the location of 1 Kelvin. Understanding electromagnetic radiation requires some appreciation of projective geometry, as each wavefront actually exhibits curvature (1/R) at all distances.

  6. Mindert Eiting says:

    ‘But, radiation in itself does not universally constitute a thermodynamic ‘energy transfer’.’

    Outline of the required proof:

    If in a closed thermodynamic system it were allowed to vary effectively amounts of back radiation (BR) over the surface, this may cause decreasing entropy (P1). This violates the Second Law and therefore in a closed system amounts of BR cannot be varied effectively. When the amounts can be varied without energy input (P2), it follows that BR cannot be effective (in any respect, including energy transfer).

    This is a deductive argument with P1 and P2 to be proven. Formally, the proof is watertight. So we need a proof of P1 and P2. This must be a piece of cake for Kristian or Will.

    • Please define what you mean by (P1), (P2), power, entropy, or energy? Also please define your term radiation, is it radiative flux (a power transfer) or radiance (“only” a potential for any such flux)? Radiative flux is proportional to the difference between two opposing radiances and only in the direction of the lower radiance.
      For thermal electromagnetic radiative flux there is no need for a closed system. Such flux (power per unit area) is independent of energy, mass, or time. A horsepower is never some dribbling of energy over time. A horsepower hour is work or entropy, never energy. Thermal electromagnetic flux is always spontaneous, and one of the few ways of dispatching entropy, (power, time, and temperature) to else where/when with lower radiance. Such flux need never be absorbed, it is a large universe. 🙂

      • Mindert Eiting says:

        Proposition 1 and proposition 2. Didn’t you understand that? Forget your definitions and prove something.

    • Where are the propositions? Why must you be a warmist giving no clue as to what you may be spouting? Why claim any closed system? There are none in this physical. You insist on some fantasy of back radiation that you wish to vary. How to vary what is not! Formally you have no proof of anything except that Newton’s laws may be self consistent.

      • Mindert Eiting says:

        ‘Where are the propositions?’ Propositions are written in my text right before symbols P1 and P2.
        ‘Why must you be a warmist.. ‘ Did I ever say that? This is bizarre.
        ‘giving no clue’. I wrote that Kristian’s proposition can be proven.
        ‘ Why claim any closed system’. You need that in the argument. False in one system implies false forever.
        ‘There are none in this physical.’ Any approximation may suffice.
        ‘You insist on some fantasy of back radiation that you wish to vary’. This is the IPCC claim to be falsified. The IPCC says that amounts of IR back radiation can increase till alarming levels. So it effectively varies in their claim.
        ‘Formally you have no proof’. If P1 and P2 are proven, the proof is watertight. There is nothing circular in the argument.

        Perhaps you do not understand that Kristian’s proposition can be proven and that a proof is something else than shouting yes or no in a fruitless debate going on for decades. The proof only uses the Second Law. No SB equation needed. Give a proof of P1 and P2 and the IPCC claim is falsified for once and for ever.

    • okulaer says:

      Mindert,

      I’m not sure I understand your suggested ‘proof’ the way you present it. Still, I believe I have already shown why the whole idea of “heating by back radiation” would directly violate the 2nd Law:

      “According to the ‘bidirectional flow’ concept, the radiative emission fluxes are simply dependent on each object’s surface temperature, so as long as the sphere’s blackbody surface stays at 255K, it will always send out a ‘radiative emission flux’ of 240 W/m2, matching the ‘heat flux’ equivalent from its internal power source. So as much energy escapes the surface of the sphere per unit time as radiation as what enters at the core of the sphere per unit time.

      So how come, as the shell warms, the sphere warms also, if its radiative output stays the same as always?

      Because as the shell warms, the q (the ‘radiative HEAT flux’ from the sphere to the shell) decreases, which means the sphere’s Q_out decreases. And according to the ‘bidirectional flow’ concept, how does the q decrease if the radiative emission flux from the sphere stays the same? By the opposing radiative emission flux from the shell increasing, of course:

      q = σ(T_sphere^4 – T_shell^4)

      (The larger the T_shell^4 term with the sphere temp kept constant, the smaller the q. Simple as that.)

      So if q needs to be preserved (and it does), then the T_sphere^4 term needs to increase correspondingly.

      But how does the sphere’s surface temperature rise? It can only happen by a direct increase in its internal energy (+U). Energy from somewhere will have to pile up.

      But from where?

      We know already that the energy in from the power source at its core is constant, it never changes. No help there. We also know that the emissive power of the sphere (its radiative output) is completely and only due to its surface temperature. In the ‘bidirectional flow’ model. So unless it actually cools, its output rate will not diminish. No help there either.

      That leaves only one alternative.

      Only the now ‘extra’ energy in from the shell is available to pile up at/below the surface of the sphere. The “back radiation” flux from the warming shell. That’s the only difference. The only ‘new’ energy.

      In the ‘bidirectional flow’ model.

      So in effect, an extra ‘heat input’ has been added to the sphere that wasn’t there before the shell came into place. A ‘radiation flux’ merely, you say. No, ‘radiative heat‘. A physical transfer of energy to the sphere, directly and all by itself increasing its ‘internal energy’, thus raising its temperature and, consequently, its corresponding ‘radiative emission flux’ out. Back towards the shell. [235+235=] 470 W/m2.

      That’s a transfer of energy as ‘heat’. By thermodynamic definition.

      From cool shell to warm sphere …”

      (…)

      “The situation with the heated central sphere and the surrounding steel shell insulating it (Eschenbach’s ‘Steel Greenhouse’) is the exact equivalent to the rGHE “back radiation” idea of how the surface of the Earth warms beyond its pure solar radiative equilibrium temperature; also a three-body setup, sun > sfc > atm:


      (…) (Derived from Stephens et al. 2012.)

      Evidently, the Sun here could itself only possibly warm the surface as far as 165 W/m2 (232K), so you need the addition of the 345 W/m2 down from the cooler atmosphere to warm it to 289K, radiating a corresponding blackbody emission flux of 398 W/m2. In other words: The entire rise in surface temperature from 232 to 289K (57 degrees) is specifically due to the absorption of the atmospheric extra radiative energy input to the surface, the DWLWIR (“back radiation”) ‘flux’, and nothing else. Note that there is NO restriction whatsoever to the outgoing radiation from the surface at any point in the cycle. As per the ‘bidirectional flow’ concept. The warming, then, is ONLY caused by more energy coming IN, not in any way by less energy going OUT. Increased energy input. More ‘heat’ to the surface. And that extra heat is NOT from the hot Sun, but from the cool atmosphere …”

      From this series’ Part 3: https://okulaer.wordpress.com/2015/01/25/to-heat-a-planetary-surface-for-dummies-part-3/

      • “According to the ‘bidirectional flow’ concept, the radiative emission fluxes are simply dependent on each object’s surface temperature, so as long as the sphere’s blackbody surface stays at 255K, it will always send out a ‘radiative emission flux’ of 240 W/m2, matching the ‘heat flux’ equivalent from its internal power source. So as much energy escapes the surface of the sphere per unit time as radiation as what enters at the core of the sphere per unit time.”

        What total Climate Clown intentional FRAUD.

        Any thermometric temperature of 255, never ever has a emissive flux of 240 W/m^2. This may be part of Real Fantasy but never part of this physical. This is but a replay of the Willis

        (‘So how come, as the shell warms, the sphere warms also, if its radiative output stays the same as always?”)

        Because as the shell warms, the q (the ‘radiative HEAT flux’ from the sphere to the shell) decreases, which means the sphere’s Q_out decreases. And according to the ‘bidirectional flow’ concept, how does the q decrease if the radiative emission flux from the sphere stays the same? By the opposing radiative emission flux from the shell increasing, of course:

        Indeed, but no clarification of process. The temperature of the sphere spontaneously increases so that the same flux can be emitted in the presence of much opposing radiance from the temperature of the shell! Please carefully examine the equations, but ignore all that your enemies claim the same equations may mean. Your enemies mean only to steal, then destroy.

      • Mindert Eiting says:

        Thanks, Kristian. I understand very well that you are right. The point of my simplistic approach, is that we only need the Second Law. SB is not really needed as far I can see. We may even postulate that back radiation is something angels or devils do. Whatever it is, it may decrease entropy in a closed thermodynamic system. That would be fatal in a basic sense.

      • Mindert Eiting says: February 21, 2015 at 3:13 pm
        “Thanks, Kristian. I understand very well that you are right. The point of my simplistic approach, is that we only need the Second Law. SB is not really needed as far I can see. We may even postulate that back radiation is something angels or devils do. Whatever it is, it may decrease entropy in a closed thermodynamic system. That would be fatal in a basic sense.”

        Ok fine, nothing is needed except your fake equations of fantasy, signifying nothing, except your intentional FRAUD.! Please indicate how any of your equations, display any correspondence to what has been measured in this physical…. Begone Satan, earthlings struggle mi-tally, each day, to overcome such Bull Shit!

  7. Mindert Eiting says: February 21, 2015 at 3:13 pm

    “Thanks, Kristian. I understand very well that you are right. The point of my simplistic approach, is that we only need the Second Law. SB is not really needed as far I can see. We may even postulate that back radiation is something angels or devils do. Whatever it is, it may decrease entropy in a closed thermodynamic system. That would be fatal in a basic sense.”

    You are correct in an electromagnetic sense as long as this universe is expanding. If ever it starts to, collapses, time to buy tickets to else where/when!

    • Mindert Eiting says:

      I would not call Newton’s law a fake equation of fantasy. Applies well at approximately closed systems. Insert in the equation the back radiation effect and see what happens on a surface when the effect varies. Violation of the Second Law tells that back radiation cannot be effective, whatever its interpretation. If you have better equations, do the same in order to see that SB is not needed.

  8. While scanning comments at WUWT
    —————————————
    Leo Smith February 23, 2015 at 1:59 am

    In one of Castenada’s novels there is a story about a young man who left his poor village in Mexico and went to the city to get an education.

    On his return to the village years later he found that the villagers were in thrall to a man who had a book, out of which he read long passages. This book it appeared contained all they needed to solve their problems. However the young man noticed that the man was holding the book upside down.

    “Your hero is a fraud: He cannot read!” he declared. “And I can prove it, he is holding the book upside down!”

    “What difference does it make, to a man who can’t read, which way up the book is?” retorted the man, and the villagers cheered …

    The problem is, that when people reject all of science already, a scientific refutation of global warming is (politically) meaningless. I too have been appalled by the standards of debate over this, and other, issues. I have come to a terrifying conclusion.

    Perhaps less than 10% of the population understands science at all, and of that 10% probably less than 10% actually understand the mathematical principles involved in the AGW proposition. And most of those are not in
    climate science.

    This is ultimately both something that has always been the case with science and indeed rational thought, and something that is deeply worrying right now, because we are in a deep crisis as a society and need better understanding than that.

    Humanity en masse proceeds along more or less bigoted lines according to the fashionable prejudices of the age. The AGW protagonists understand this: Their business is to move the fashionable bigotry along to suit their agenda.

    If we step back a minute and regard the implications of what I propose to be the case, they are these: The vast majority of humanity is incapable for one reason or another of understanding the science and technology that forms the backdrop to their lives. And in a democracy that means they are more or less unfit to vote on matters that affect it.

    A small minority of ‘movers and shakers’ – and these days they are (to borrow Jilly Coopers terminology) the ‘Tellystocracy’, the media luvvies and those who use mass media to ‘inform’ public debate – are the ones who count. They are the new elite, the new lords and masters of the brave new world, and it is this group that has been so thoroughly targeted and infiltrated by all and any group with a political or commercial axe to grind. It doesn’t matter what some obscure group of scientists believe, or what the mass of people believe, what matters is what this group do in terms of forming (rather than informing) public opinion.

    This group then are by and large the group that actually carries out political change. They are in charge of the fashionable bigotry that comprises what we have come to know and love as political correctness. That vast and loosely affiliated propaganda machine that tells us what to think about, and what to think about it.

    What we need to do if we are to introduce truth into this tissue of lies and deceit, is to make the case to the media/political luvvies that in fact their particular brand of bigotry is deeply dangerous to themselves as a class.

    In the case of AGW we have two main avenues through which this is happening.

    First of all the man in the street is getting fed up with falling standards of living, and his winters seeming just as cold wet and miserable as the summers are, despite claims it was the warmest year on record.

    Secondly the more astute members of the tellystocracy are becoming aware that infrastructure is for everyone, and that includes them. Victorian sewers were to protect the elite of the day from disease, by eliminating it from the great unwashed. This is a potent line of attack – Wind turbines and solar panels become not source of individual profit, but a disaster for all including those that profit from them.

    Ultimately the game is this: Science in its broadest terms is nothing more and nothing less than a means of predicting the future. Science says if we do this or that, the other will happen. The complex mathematical laws we deduce, infer or discover (according to your metaphysical picture of what Laws are) have no justification beyond the fact that they work, and what they say will come to pass, comes to pass, mostly.

    Science that fails to predict anything is untestable, and if it fails to produce the result that reality provides, it’s junk science or no science at all. You can summarise this by saying that in the long term reality trumps bullshit.

    Ultimately AGW either produces correct predictions or its junk, It’s looking to be junk. However that doesn’t stop people believing in it because it’s fashionable bigotry. But here we invoke Darwin. Societies that fail to realise what reality is, and cling to fashionable bigotry, will suffer accordingly. There are signs that the whole West will in fact ultimately collapse in an orgy of self destructive mutual deception and liberal angst. Or perhaps it will wake up and smell the coffee.

    And in the end, that is the conundrum. It is true to say that people are reasonably easily led, and that even those that lead them, are themselves subject to fashionable bigotry. That is a fact of life that we have to deal with. In the end we have only one yardstick that works to dispel the fog of Belief In Bullshit and that is Reality herself, and Reality is a hard mistress. If She needs to destroy entire societies that are so infected with irrational bullshit that they can no longer support themselves at all, She will.

    I don’t like to get political here, but this is to me the great argument for not having the sort of monolithic world government that the cultural Marxists of the UN and the ‘liberal and social’ democracies seem to espouse. that and we all go down together. Whereas having political islands of national ideologies at least allows for some diversity of political thought, and if the West becomes so decadent not because of Capitalism, but because of Marxism and its descendants itself, that it is in danger of falling to a stronger culture, maybe one of those political islands will have the tools and the strength to resist and prove to have the next line of fashionable bigotry to deal with the new reality.

    From my perspective there are two completely different dimensions in play here, and it helps not to confuse them.

    There is the technical and scientific reality of the data: That the world ain’t warming any more, never warmed very much, and windmills and solar panels are a complete waste of time and money, and destructive to boot, and if we want to stay alive in the absence of fossil fuel the logical alternative is nuclear power.

    That these things are provably and demonstrably true is, however, irrelevant to the second dimension, which is what people think. Or can be induced to believe. And here there is in fact a world war in progress, World War III. Its not being fought with weapons (much) that kill, directly, but with weapons that corrupt thinking. It is a war of propaganda and competing ideologies, none of which have a particularly strong basis in Reality, because Reality is pretty damned complicated, and its easier to get people to believe in simple stuff. ‘Four legs good – two legs bad’ sort of stuff.

    I have to say that I have more or less given up on the science: The jury is in for people to understand the maths and the physics and how real science works. AGW is a crock of shit, and that’s that.

    The real game is the war for hearts and minds. And that is a game of psychology, propaganda, money, power, politics, greed, fear, uncertainty and doubt. If we can’t win it, it will in the end destroy Western civilisation, and so it should. If we have no answer for lies, we don’t deserve to make it.

    Once we had a system that worked. The brightest and best, and a few of the rich, got excellent educations and were indoctrinated with a culture of care for those less fortunate, and a sense of duty towards the masses. They did what they considered to be right, after duly listening to the problems.

    Today that is destroyed by egalitarianism, which ensures that no one at all gets a good education that everyone cannot afford. Except for a very very few – too few – people who espouse state education but manage to avoid it in the case of their children. Worse, they dont educate them into the actualities of science and technology even then, they educate them into the practical techniques of propaganda. We have in short a generation of peole who are highly skilled in the manipulation of public opinion, but no idea how a smart phone works. People ideally placed to control and dominate a society, and take from it all its riches, but without actually having even the most basic understanding of how those riches are created.

    Such a situation is dynamically unstable. We, the technologists, are not screaming out for recognition ‘because its unfair’ or ‘because its morally indefensible’. No, we have a much quieter but devastatingly powerful message: “If you don’t take at least some notice of Reality, you will in fact die of ignorance, and likely take us with you.”

    *shrug*

    If they don’t listen, it’s Goodnight Vienna. We won’t be the first culture to commit racial suicide in pursuit of idiotic beliefs.

    —————————————-

    dbstealey February 23, 2015 at 2:56 am
    Leo Smith, You’re one in a million! Thanks for posting that, I’m in complete agreement. The problem isn’t science; that is 100% on the side of skeptics of MMGW. The problem is human nature. Some people/groups have that figured out, and that’s bad news for the rest of us.

    A.D. Everard February 23,2015 at 4:30 am
    Leo, your comment should be a post in itself. It should be widely read. This is so much what needs to be understood. Thank you.

    George Tetley February 23,2015 at 12:49 am
    WOW !!!

    • okulaer says:

      Thanks for that, Will 🙂 Ever so slightly off topic, but thanks anyway.

      I’ve taken the liberty of rearranging your comment a bit so as to make it more readable.

      No doubt this guy hits the nail …

      • Kristian,
        I thought you might enjoy the clarity (political) expressed by Leo. However, Leo gives me no hope of some political correction! Perhaps some of the 10% of the 10%, can somehow get their shit together, and actually admit “I do not know”, but can actually demonstrate that the greenies know much less, but only demand that all unconditionally believe “their” nonsense.
        Then perhaps such can generate some general aversion to, “die of ignorance, and likely take us with you.”. I know you dislike my choice of words as much as I dislike yours. 🙂 The question is how to form bullet proof written statements of this physical (is), with admittance of (don’t know), that will be accepted by the 90% pitchfork and torch bearers, on the side of survival! The skill set is the ability to think, and to also use the ability of others to think differently. Humm maybe!, coupled with survival is most attractive to me.

  9. Rafael Molina Navas says:

    To be honest, I must say I find you very smart … especially in juggling with concepts, words, and even with animated figures.
    ALL events are “just potential (hypothetical)” (not only radiation energy transfers) before they actually happen … But not beeing English my mother tongue, I´m not going to discuss with you on that field.
    But as radiation from the cold object in the direction of the hotter is real, ITS energy must also be real, not hypothetical. What happens with that energy when that radiation reaches the hotter object, if its transference is “forbidden” ?
    Or with your animated figures are you suggesting that the electromagnetic waves in the direction of the hotter object are canceld out by what coming in the opposite direction?

  10. Voidness says:

    Quick question.

    Regarding the standing waves.

    Since a thermal source has a limited spatial coherence in the emitted waves, black radiation has coherence lengths (lc) of about lc*T ~ 1e-3 mK for instance. How can there be standing waves between two, by radiation interacting thermal sources?

    • okulaer says:

      Hi, Voidness 🙂

      I can’t seem to see immediately how coherence length has anything to do with whether or not opposing waves of equal wavelength (and amplitude) from different sources interfere when meeting. Real objects emitting blackbody radiation (visible, thermal) are indeed incoherent sources, but individual EM waves emitted from one object at various times from various places/atoms will still meet, overlap and interfere with opposing individual waves emitted by a facing object. This is definitely a matter of statistical, probabilistic distribution.

      But maybe I’ve misunderstood you. You bring up an interesting point. Could you perhaps elaborate a bit on your position?

      • Voidness says:

        “I can’t seem to see immediately how coherence length has anything to do with whether or not opposing waves of equal wavelength (and amplitude) from different sources interfere when meeting.”

        It is late here now, so forgive me if what I write seems strange in any way.

        My spontaneous thought is that for that kind of interference you need spatial coherence at least on the scale of the separation between the sources. Otherwise the disturbances just pass through each other in the intermediate space. In the 2D animation above the two sources are in phase, separated around 2 wavelengths and has infinite spatial coherence. I would claim that is a fundamentally different situation than what we are discussing here.

        One other possibility would be that when a disturbance reach the other body, it will interfere with the creation of a disturbance. But this seems implausible for room temperatures and above at least (probably also for approaching zero Kelvin as well). Room temperature (lc ~ 10^-3/300 = 10^-5m) would indicate a typical temporal coherence of lc/c ~ 10^-14s. The fastest time scale for one oscillation in a solid are on the order of 10THz, one order of magnitude slower then the passage of a disturbance.

        Another thing that just crossed my mind is that the polarization in a thermal source is randomized, that should also make it harder for interference to actualise. Am I right?

        If I’m correct, this indicates that the wave character is of no importance and the problem is purely a ballistic one.

      • Voidness says:

        “No, the ‘photon’ is simply an individual light wave, one distinct EM wavefront. It is the wavefront itself that moves forward at the speed of light, not something moving along its length.”

        From the more sophisticated description, a photon is an excitation in a quantum field, the photon field. The photons are modelled as bosonic excitations of the photon field and are then a collection of quantum harmonic oscillators and as such have angular frequencies. We can also work with other formulations, besides this canonical approach, as the path integral approach. But let’s not go there, it doesn’t really shed light 🙂 and clarity on the issue.

        Besides that, since the electromagnetic field carries both energy and momentum, as any wave, the complete specification should include both frequencies (energy associated with a plane wave) and wavevectors (momentum). The wavevector is connected with the wavelength as the magnitude of the wavevector, |k| = 2\pi / \lambda. Any reasonable disturbance can then be expanded in a plane wave basis.

        You obviously have experience with equilibrium thermodynamics, but from the speculations about light here, and forgive me from doing so, I must guess that you are not a working physicist. Chemist or mechanical engineer maybe?

        • okulaer says:

          You’re right, Voidness, I am not a ‘working physicist’. And frankly I see that as an advantage. Because people specifically trained and specialised within a certain scientific field tend to think very much within the confines of that field when approaching a topic touching upon their realm of expertise. There are certain deep-seated paradigms that keep us kind of boxed up inside certain mental frameworks. I’m not saying that you are necessarily like this (I don’t know you), and I’m also not saying that I’m not affected myself.

          What I’m trying to say is that, from how I read what you write, Voidness, you seem to be somewhat missing the point here. The arguments you put forward – as pointed and well-reasoned as they may be in themselves – simply hit a bit wide of the mark. I know this sounds like a cheap attempt to evade the issues you raise, but bear with me.

          You’re focusing on specifics, which is fine. I guess. But the scope of my model is fundamentally general. It addresses stochastic, probabilistic averages of a mind-bogglingly vast number of superimposed electromagnetic fields. When I talk about an ‘individual wave’, I am not referring to one single specific wave among trillions. I am referring to the ‘average’ of all similar waves. The ‘average’ individual wave of, say, a particular wavelength.

          It serves no meaningful purpose, the way I see it, trying to track down (physically or theoretically (mentally)) every singular ‘photon’ in a radiative heat transfer to examine its different properties (direction, phase, wavelength, coherence, polarisation) and how these might influence its chances of interfering with other ‘photons’. It can’t be done. And it doesn’t have to be done …

          You say for instance: “My spontaneous thought is that for that kind of interference you need spatial coherence at least on the scale of the separation between the sources. Otherwise the disturbances just pass through each other in the intermediate space.”

          And: “Another thing that just crossed my mind is that the polarization in a thermal source is randomized, that should also make it harder for interference to actualise. Am I right?”

          You are right. At a ‘uni-photonic’ level. Any one distinct ‘photon’ (or wave train pulse) emitted by random excitations in any random atom/molecule at the surface of the one multidimensional thermal source in a radiative heat transfer, at any random time, at any random wavelength, in any random direction into the 3D space abutting it, and in any random state of polarisation, will have the definite possibility of travelling undisturbed all the way to the opposing thermal source. So you can always object to my general (and, yes, rather crude) model, saying: ‘No, what you’re describing can’t be right, because individual ‘photons’ this and that …’ And I can’t really argue against you if you do. And I won’t.

          The point is this: Such an approach betrays a mindset ultimately stuck in a macroscopic, linear, classical-mechanical (Newtonian) view of the world, where ‘photons’ are still conceived of as somehow flying independently around like ping pong balls down separate highways through the emptiness of space. And I can certainly sympathize with this view. The mind flounders, trying to make some sense of something so strange and unfamiliar (and invisible at that), that it can never really fully grasp it. So it yearns for some kind of anchor point. And grabs hold of what it already knows. But this approach will ultimately lead nowhere – if anywhere, only to confusion and frustration. Because if applied to a fundamentally and profoundly probabilistic problem like this one, it will forever remain vastly inferior to a more straightforwardly stochastic analysis, made suitable from just that inherent randomness that pervades, on the lowermost level, every aspect of blackbody radiation.

          It is only when zooming out that the seethingly bewildering and disturbingly haphazard nature of the quantum world slowly starts merging into certain patterns. There’s an overall structure appearing, a certain order to the madness. It all starts making sense. And the mind rejoices. Grouped together, things aren’t as coincidental as one was perhaps led to believe.

          We simply do not and cannot know what a single ‘photon’ is up to inside a radiative thermal exchange. So why bother trying to pretend as if we did? We don’t know if our conceptual idea of a ‘photon’ travelling through space is even real. If it exists as a separate entity at all. We cannot experience ‘photons’ as quanta of light, after all, until light somehow strikes matter. We cannot physically pinpoint discrete ‘photons’ or electromagnetic fields inside a radiative thermal exchange. And we don’t have to. Because whatever their individual behaviour and the effects of their individual behaviour, they always average out to macroscopically observable phenomena, like transfers of energy.

          Being unable to observe interference patterns (as with incoherent light (e.g. thermal) sources facing each other) doesn’t mean they’re not there, that interference doesn’t happen. It simply means that the superimposed field changes so fast at any point in time and space (on the microscopic level) that our eyes or instruments cannot possibly keep track. Individual conditions for interference in any one place are only maintained for exceedingly short time intervals. But what is the probability of the average particular wavelength ‘photon’ being emitted from the first blackbody interfering with a similar ‘photon’ being emitted from the second one? Not of any single ‘photon’. But of the average ‘photon’ of that wavelength.

          The way I see it, this would naturally even out to a situation where one can simply consider the two blackbody emission spectras as a whole facing each other, leaving only the net ‘transfer’. (So we’re essentially back to the macroscopic ‘heat’.) Every average ‘photon’ would find an equal opposing partner to hook up with. Except for the ‘surplus photons’ from the warmer object. This is what my model is trying to express. I prefer to place myself somewhere midway between the fully microscopic (quantum/photonic) and the fully macroscopic (thermodynamic) perspectives on this process in order to interpret it.

          I must emphasize that I am not in any way claiming to have finally pinned down the Truth about this problem. I’m not even saying that my model is necessarily awfully accurate when it comes to describing quantum-theoretical matters. I am not trying to topple or replace centuries of accumulated knowledge on the properties of light. I am rather trying to combine this knowledge of the microscopic with the macroscopic effects as observed and explained by thermodynamics.

          I obviously don’t know what’s really going on in there. And I submit that no one does …

          What I do know is that you can’t explain what happens in a radiative heat transfer with a ‘bidirectional transfer’ model. Because this ends up forcing you to violate the 2nd Law of Thermodynamics. The warmer object in a radiative thermal exchange process simply cannot be understood to absorb for energy gain the radiation from the cooler object in that same process as a separate energy flux (transfer/input), basically as ‘heat’.

          Radiation and radiative transfer do not violate the 2nd Law of Thermodynamics. Neither are they exempt from it. They are completely constrained by it. Like everything else in the Universe.

          – – –

          So how do you explain ‘radiative insulation’, a cooler object reducing the cooling rate of a warmer one?

          That’s the issue at hand.

          The traditional way of describing this process is by saying that ALL energy emitted by the cooler object reaches and is absorbed by the warmer one, raising its internal energy [U], but that ALL energy simultaneously emitted by this warmer object also reaches and is absorbed by the cooler one, so that the net result of this perfect, undisturbed exchange is an absolute increase in the cooler object’s U an an equal reduction in the warmer object’s U. So not one single path of one single ‘photon’ from either object to the other is ever obstructed or interfered with in any way. The two objects are both separately perceived as pure emitters to the opposing one, radiating freely and unrestricted at the facing object, its opposing presence as inconsequential as an infinite, perfect vacuum would be.

          In this world view, then, there isn’t really any energy transferred as ‘heat’ at all in a radiative heat transfer. There is only a mutually independent two-way transfer of energy in the form of electromagnetic radiation. And what we call the ‘heat transfer’ is then simply the net result of this radiative exchange – more radiation is absorbed by the cooler object than what is absorbed by the warmer one. And this circumstance alone constitutes ‘the heat’ …

          So the radiation effectively heats at both ends of the exchange. There is in a sense ‘a double heat transfer in one’: All the energy transferred in opposite directions by electromagnetic radiation inside this single ‘radiative heat transfer process’ is absorbed and thus raises the U and consequently the T at both the warmer and the cooler end. And this is specifically what traditional (thermodynamically defined) ‘heat transfers’ do. There is no fundamental distinction between the two energy transfers in this regard. The warmer object simply ends up cooling more than it heats and the cooler object ends up heating more than it cools.

          And that’s how the ‘bidirectional transfer’ explanation goes …

          To me, a hardcore ‘thermodynamicist’ (or whatever it’s called), this description of reality screams ‘ill-considered!’ and ‘contrived!’ at the same time. The whole notion of an energy transfer from a cool to a warm object being offhandedly allowed to heat directly (increase the internal energy content of) that warm object, the positive temperature effect of which is only prevented by a bigger and parallel cooling effect, doesn’t sit particularly well with my general instincts. To put it mildly. Alarm bells ringing.

          Still, my instincts could be wrong of course. At this stage, the 2nd Law does hold, after all. At least outwardly. In the ideal situation where the warm object involved in the heat transfer actually and naturally cools (its temperature decreasing in absolute terms).

          The real problem with the ‘bidirectional transfer’ model arises only when the warm object is no longer naturally cooling, but is kept at an equilibrated steady state temperature by the constant energy input from an external/internal heat (power) source.

          At this point, our first radiative heat transfer process will end up making the temperature of the warmer object actually rise in absolute terms. Its temperature will no longer simply drop more slowly. It will actually rise.

          And the question then becomes: What energy is it that makes the temperature of the warmer object rise? And where is this energy coming from?

          These are highly relevant questions for anyone interested in the 2nd Law of Thermodynamics and what it dictates. And we all bloody well should be interested. In one way or another. Without it, there would be no Universe.

          A close inspection of the various energy flows in this new situation, based strictly on the principles of the ‘bidirectional transfer’ model, can only lead us to one conclusion: The temperature of the warmer object rises solely as a result of the absorption of the (‘extra’) energy directly transferred to it from the cooler object.

          In other words, the direct heating effect of the warmer object resulting from the absorption of the radiation from the cooler object – which was always there, but simply concealed before by the bigger cooling effect from the simultaneous emission from the warm object itself – can no longer be hidden.

          If there is one thing the 2nd Law tells us, however, it is that you cannot naturally have a transfer of energy from a cold place to a hot place directly and all by itself making the hot place even hotter. There is no way that a single heat transfer process could ever make the absolute temperatures (energy levels) at BOTH ends of the process higher.

          But, you might argue, when there is a second heat transfer process involved, like in this latter case, then the heated and insulated warmer object will necessarily warm further. It is what we observe.

          Indeed it will. The EFFECT is very real. But you cannot EXPLAIN the effect by the cooler (insulating) object directly heating the warmer object some more, on top of the original heating by its actual heat source. An insulating layer does not work by increasing the HEATING of what it insulates. It can only ever reduce the COOLING of the insulated object.

          And you do not reduce the cooling rate of an object by increasing its heating rate. Those are two completely different (in fact, opposite) processes. This is a pretty basic principle of nature. And, I would hope, a rather intuitive one at that …

          Remember, the electromagnetic energy coming IN can not do anything to ever hinder the electromagnetic energy going OUT from going out. In the ‘bidirectional transfer’ model. So there is no way the radiation from the cooler object can prevent the energy from the warmer object’s heat source from escaping the warmer object again through radiation. The incoming radiation from the cooler object does not in any way impede the cooling of the warmer object. In other words, it is not the energy from the heat source that piles up when the warmer object starts warming beyond its initial equilibrated temperature. Only the addition of extra energy to the warmer object from the cooler object. This is the energy that raises the U and the T of the warmer object in absolute terms, directly and all by itself, and that thus forces the warmer object to emit more radiation, not less, the energy in>out from the heat source PLUS the energy in>out from the cooler object, in order for it to increase its radiative cooling rate to balance its increased heating rate.

          In my book, this is a totally absurd way of decribing the internal workings of a radiative heat transfer …

          And this is where, in my mind, the ‘bidirectional transfer’ EXPLANATION fails.

          – – –

          What I am proposing, then, is simply an alternative model where the radiative ‘exchange’ occurs rather in the space between the two thermally radiating objects and where the actual transfer of energy is spontaneously realised as a UNIdirectional ‘net’ flow from the warmer to the cooler object, through the integrated/superimposed radiation field between them. How exactly this happens in detail is anyone’s guess. But in this model, at least, the cause of the ‘extra’ rise in temperature of the warmer object would be the further piling up of energy from its actual heat source, not from its insulating layer, its cold reservoir; the energy coming IN from the source is simply no longer able to escape the surface of the warmer object as fast as before the insulating layer was introduced.

  11. Rafael Molina Navas says:

    AS I HAVEN´T GOT ANY ANSWER to my post of last 14th, I repeat here what I´ve recently posted on another related blog:

    You haven´t replied my six days old post … But I can read now, on your answer to Jerry:
    “I’ve explained why the energy from the cooler atmosphere cannot be absorbed by the warmer surface (because that would ultimately lead to a violation of the 2nd Law)”
    You have been told by some of us that your interpretation of 2nd law “may” be wrong … So, beeing that the core of the issue, what quoted CAN´T be considered any explanation whatsoever.
    You also say:
    “…radiative energy can only be transferred through space by the movement of EM wavefronts, so when a set of wavefronts is no longer able to move separately in a forward direction (like when it meets a more energetic set of wavefronts), then the radiative energy it carries is also no longer able to move any further in that direction”.
    Do you actually think so? Do you REALLY mean that electromagnetic waves in opposite directions, even of different frequencies, can cancel each other out, and their energies kind of disapear?
    I´m not an expert, but feel pretty sure that is utterly wrong. I can understand now why perhaps you didn´t unswer my post, where at the end I already asked:

    “Or with your animated figures are you suggesting that the electromagnetic waves in the direction of the hotter object are canceld out by what coming in the opposite direction?”

    • okulaer says:

      Rafael, you say:

      “You have been told by some of us that your interpretation of 2nd law “may” be wrong … So, beeing that the core of the issue, what quoted CAN´T be considered any explanation whatsoever.”

      This is very simple. What we are all trying to do is explain an observed EFFECT. The effect is real. It violates no laws of physics. Of course it doesn’t. It’s a physical effect, governed by those very laws of physics. We observe what we observe.

      The observed effect is this: The rate at which energy in the form of ‘heat’ is transferred (the intensity of the ‘heat flux’) from a high-temperature object to a lower-temperature object decreases progressively as the difference in temperature between the two objects grows smaller and smaller. In the end, when the two objects have ideally reached the same temperature, the transfer rate (the flux intensity) has dropped to zero. The thermally induced transfer of energy has ended.

      This is ALL we physically observe. Everything beyond this amounts to no more than interpretational descriptions of the situation, the construction of mental models drawing up invisible processes thought to be at work.

      These descriptive models are purely theoretical, conceptual ideas, not physical reality itself. I will repeat the words of Niels Bohr once again: “There is no quantum world. There is only an abstract quantum physical description. It is wrong to think that the task of physics is to find out how nature is. Physics concerns what we can say about nature …”

      Rafael, it is not the observed EFFECT that violates the 2nd Law of Thermodynamics. Nobody claims it does. It’s there. It’s real.

      No, what ends up violating the 2nd Law is only the theoretical attempt at EXPLAINING a radiative heat transfer process through the ‘bidirectional model’, that is, by saying that energy transfers both ways, from hot to cold and from cold to hot, only more from hot to cold, and that – as a natural consequence – hot also absorbs the energy transferred from cold.

      I’ve explained ad nauseam why this ultimately cannot be. For instance here:

      ‘To heat a planetary surface’ for dummies; Part 3

      When your attempted explanation of how an observed physical effect comes about ends up violating the 2nd Law of Thermodynamics (again, not the effect itself, only your explanation of it), then your explanation is wrong. Simple as that. Back to the drawing board. Try something else.

      And that is what I’m doing here. I’m not saying “I’ve now got the Truth forever pinned down!” Not at all. Mine is also merely a theoretical model. An attempt at conceptually explaining the observed effect described above. In a thermal process, we only ever observe/detect the energy transferred as ‘heat’. A strictly UNIdirectional transfer. We cannot possibly tell what’s really going on at the quantum level inside this macroscopically observed heat transfer process. We can only hypothesize …

      The point is only that my (UNIdirectional) model doesn’t violate the 2nd Law. The bidirectional one does.

      “Do you REALLY mean that electromagnetic waves in opposite directions, even of different frequencies, can cancel each other out, and their energies kind of disapear?”

      Not at different frequencies (wavelengths), Rafael. You need to actually read what I’m writing. Opposing waves of different wavelengths cannot create a standing wave pattern. That’s part of the definition.

      And a standing wave pattern is not about making the energy of oppositely moving waves magically ‘disappear’. The energy is of course still there. It simply cannot move through space. There is no net propagation of energy in any direction in a standing wave pattern, Rafael. But energy doesn’t disappear simply because it doesn’t propagate.

      • Rafael Molina Navas says:

        Thank you.

        You say:
        “Not at different frequencies (wavelengths), Rafael. You need to actually read what I’m writing. Opposing waves of different wavelengths cannot create a standing wave pattern. That’s part of the definition”.
        But radiation coming from different objects and at different temperatures have not necessarily the same frequency … And even with same frequency, waves should be exactly in opposite phases to cancel each other out …

        You also say:
        “And a standing wave pattern is not about making the energy of oppositely moving waves magically ‘disappear’. The energy is of course still there. It simply cannot move through space”.
        Even forgetting what previously said, “your” theory woukd have an important loose end: in what form that still energy could be “there”?

        By the way. I contacted Feldman and sent him the link to your challenging blog, asking for his opinion about it. Anfortunatelly he seems very busy, but he said:
        “Thanks for your interest and I appreciate you sending these to me.
        Kristian’s blog is very detailed and he has thought about these issues for a while. That being said, our study was focused on the effect from CO2 alone, which we found pushes the system towards a warmer state, by radiative forcing, and that blog post appears to conflate what we found with a large number of other effects. It is important to note that while there are certainly other feedbacks in the climate system, the forcing from CO2 is largely independent and separable from these. The feedbacks on CO2 forcing tend to enhance the effect of rising CO2, as cited in the paper, so I think Kristian has a sign issue there.
        It would take a significant amount of time to formulate a detailed response to every claim made in the post (which unfortunately I don’t have time for), but suffice it to say, there are numerous issues there.
        For example, I should note that the ARM observations are superior to the CERES products for surface forcing, because we’re measuring at the surface and CERES is a satellite instrument. I applaud the CERES team for putting together a surface flux product, but the surface stations have direct measurements. Also, we are looking essentially continuously at two sites rather than how CERES observes these sites occasionally and at the same time of day (CERES is sun-synchronous). Also, we have independent, in situ measurements from weather balloons, and are able to identify clear-sky scenes with very sensitive radar and lidar measurements, which are not available for the CERES products.
        Hope that helps a little. Right now, I have numerous projects with pressing deadlines, but I would be happy to respond to peer-reviewed criticism”.

  12. Voidness says:

    Thank you very much for that lengthy reply. I get the feeling that you are truly genuine in your thoughts on the matter. That is much appreciated!

    I do work with transport theory, but exclusively in solids, never radiation (I work with converting heat to electric energy through thermoelectricity). So it is exciting to try and understand this problem, quite intriguing 🙂

    I have many thoughts on the things you have written. The one most pressing is related to the following.

    “What I do know is that you can’t explain what happens in a radiative heat transfer with a ‘bidirectional transfer’ model. Because this ends up forcing you to violate the 2nd Law of Thermodynamics.”

    How do you show this? My own calculation shows no violation.

    Say you have a black body, temperature TH (for hot) and impinging black radiation TC (for cold).

    The change of entropy in the body due to energy change is

    a*1/4(TC^4 – TH^4)/TH.

    The change of entropy in the radiation field is

    a*1/3(TH^3-TC^3).

    Total change in entropy

    a*1/4(TC^4 – TH^4)/TH + a*1/3(TH^3-TC^3) >= 0, for all TC and TH.

    That is no violation, the sum is strictly positive unless TC=TH, and then the process is reversible, which seems reasonable.

    • okulaer says:

      I appreciate your interest and your input, Voidness.

      But again I must stress: It is not the EFFECT that violates the 2nd Law. It is only the way you EXPLAIN how this effect comes about with a ‘bidirectional transfer’ that does. And this is actually quite a subtle distinction that is not so easy to show mathematically. Because you always end up like you do here, showing that the EFFECT doesn’t decrease total entropy.

      That is why I prefer to use words to describe exactly what – in my mind – is wrong with the ‘bidirectional’ picture. Just because an observed effect doesn’t violate any physical laws (why would it? we observe it happening, after all), it doesn’t mean we are thereby free to explain it any way we like. We have to comply with certain general rules and principles.

      What I am trying to show is how the ‘bidirectional transfer’ model in effect claims that the ONE radiative heat transfer observed between two radiating objects at different temperatures is actually made up of TWO separate heat transfers operating simultaneously within the same integrated radiation field, strictly independent from and opposite to one another.

      This, I submit, gives the bizarre result (when you connect the warmer object to a constant heat source) that the cool insulating layer (the ‘cold reservoir’/’heat sink’ of the warm object) directly and all by itself is allowed to increase the internal energy [U] of the warm object, and hence its absolute temperature T, by transferring energy (effectively as ‘heat’) to it. This is how the (extra heating by) atmospheric “back radiation” version of the rGHE is explained:

      The surface of the Earth here moves from 232K to 289K solely because of the ‘extra’ energy (heat) input from the cooler atmosphere. The surface is only able to reach a radiative output of 398 W/m2 (from a temperature of 289K) because it allegedly absorbs an extra input flux of energy of 345 W/m2 from the atmosphere, in addition to the solar flux (165 W/m2). The atmospheric ‘flux’ is treated as if it were exactly equivalent to (giving the same qualitative result as) the solar HEAT flux.

      In other words, the problem here is NOT the fact that, upon being insulated by the cooler object placed (ideally) around it, after first having equilibrated to a steady-state temperature with its external/internal power source constantly heating it, the warm object starts warming beyond this initial steady-state temperature. That’s not the problem. That will happen. That’s the insulation effect. The problem, as I see it, lies solely in trying to EXPLAIN this effect by saying that it is in fact the cooler insulating object that directly heats it some more.

      Like I pointed out in my last comment: Insulation never works by HEATING the object it insulates. The insulating layer is NOT an energy source to the insulated object. To me, this is a universal principle. Insulation ONLY ever works by reducing the COOLING of the object it insulates. And heating some more (increase the energy INPUT) is not the same as slowing down cooling (reduce the energy OUTPUT). These are fundamentally different processes. I am sure you agree …

      Ultimately and admittedly, this whole thing might well boil down to a pure philosophical question. I come from a thermodynamic background, and in thermodynamics, an ‘energy transfer’ comes either in the form of ‘work’ [W] or ‘heat’ [Q] and it is invariably UNIdirectional. It affects directly the receiving system (increasing its U). Unidirectional energy transfers are all we ever actually observe/detect in nature. A ‘bidirectional transfer’ between two objects is a purely theoretical concept, it is never – and could never be – actually observed. A transfer of energy as ‘heat’ [Q] between two objects occurs spontaneously as soon as the one object is warmer than the other one. This is my fundamental philosophical approach – my jumping-off point – to this subject. I am simply a bit of an atheist when it comes to the idea of a ‘bidirectional transfer’. Why believe in it when all we ever observe are UNIdirectional transfers? Why complicate what Nature is telling us, showing us?

      In my mind, this problem only arose with the AGW hypothesis (based firmly on the rGHE hypothesis). Its core claim is this: Inject IR-active substances into the atmosphere, and the surface underneath will warm, just from them being there. Because they make the atmosphere ‘radiate (back) at’ the surface. I shout an emphatic “NO!” The atmosphere makes the surface warmer simply because it itself is warm, warmer than space. It is able to warm. It is also heavy. It has a weight. Pressing down on the surface. Space doesn’t. The atmosphere INSULATES the solar-heated surface. It doesn’t ‘heat it some more’. Its presence makes the escape of the outgoing energy from the surface harder. And it does so through its MASS. Not through its radiative properties.

      This will be the topic of my Part 5 in this series …

      • Voidness says:

        “I come from a thermodynamic background, and in thermodynamics, an ‘energy transfer’ comes either in the form of ‘work’ [W] or ‘heat’ [Q] and it is invariably UNIdirectional. It affects directly the receiving system (increasing its U). Unidirectional energy transfers are all we ever actually observe/detect in nature.”

        I would agree to this, under very restrictive circumstances. It’s easy to give a counterexample.

        Take an extended object, Joule heat it at one end and Peltier cool it at the other so you create a temperature gradient that reaches over the ambient temperature. Now you have a system that releases energy to the surrounding at one end and achieves energy from the surrounding at the other. That is clearly not unidirectional.

        • okulaer says:

          “That is clearly not unidirectional.”

          Of course it is, Voidness. Those are two distinct UNIdirectional transfers of energy you’re describing there. The heat transfer process moving energy from the surroundings to the one (the cool) end of your extended object is strictly spatially separate from the heat transfer process moving energy from the other (the warm) end of your extended object to the surroundings.

          A radiative heat transfer is claimed, in the ‘bidirectional transfer’ model, to be made up of two separate, opposing heat transfers occurring inside the very same field, they occupy the exact same volume of space, moving straight through one another.

          This is not at all what you’re describing …

  13. jerry l krause says:

    Hi Kristian,

    This is a response to your comments of 3/22/15 at 3:08PM and 3/22/15 at 2:41PM

    I’m taking a break from working on my income tax filing. So quickly scanned your recent comments here. I wonder when Bohr made the statement which you quoted. I wonder this because I am sure he was aware that his planetary model of the atom, where the electron(s) orbited the nucleus along quantized orbits (because the electrons had quantized energy levels), was totally removed from any reality. For as I told my chemistry students, it could not explain a snow flake. So, if the statement was made before Schrodinger made the fundamental leap that matter (electrons) might behave as a wave in certain circumstances so that Bohr’s hydrogen atom could be ‘solved’ with the wave equation. But it was de Broglie who first suggested electrons (particles) might behave as light (a wave, pure energy?). When Schrodinger made this fundamental leap, he really did not know where it would lead. Whereas, Bohr purposely quantized the orbits of the electron because he knew that the hydrogen atom produced a visible line-spectra and not a continuous spectra. And Einstein questioned the quantum mechanics involved in Schrodinger’s wave mechanics solution to the hydrogen where the electron was no longer a particle but instead had a probability that it might exist in certain volumes of space about the nucleus. Yet, it was Einstein who had to invent the photon (a particle of light which was previously a wave) to explain observations involving the photo-electric effect. So the quote of Bohr was made during an extremely revolutionary period of time of the history of physics.

    And leading the revolutionary pack were classical physicists, not someone who had little knowledge (or experience) in classical physics. These classical physicists were forced to consider revolutionary ideas because they were unable to explain, with their classical physics, the new observations that were being made by the experimentalists.

    Previously I have reviewed the fact that Pauling had concluded that the nuclear physicists knew nothing because they had invented a language that only they could speak. But in doing this he overlooked the fact that some of the same nuclear physicists he was criticizing had 20 years earlier designed a bomb that worked the first time it was tested, the second time it worked and did what it was made to do, and the third time it worked and did what it was made to do. With a record like that, how can anyone claim that these nuclear physicists didn’t know anything? And there were many other scientists and engineers who were doing what had never been done before. For we know, half a century later, that it is still a very challenging, technical, job to produce the vital ingredients required for the bomb.

    Just some thoughts.

    Have a good day, Jerry

  14. Voidness says:

    “Of course it is, Voidness. Those are two distinct UNIdirectional transfers of energy you’re describing there. The heat transfer process moving energy from the surroundings to the one (the cool) end of your extended object is strictly spatially separate from the heat transfer process moving energy from the other (the warm) end of your extended object to the surroundings.

    A radiative heat transfer is claimed, in the ‘bidirectional transfer’ model, to be made up of two separate, opposing heat transfers occurring inside the very same field, they occupy the exact same volume of space, moving straight through one another.

    This is not at all what you’re describing …”


    1.

    First a quick question, I’m not exactly sure that I follow you here. What do you mean by a field in this case? The electromagnetic field?


    2.

    I may have misinterpreted you thoughts based on how you express the first law, please correct me if I’m wrong. The

    \Delta U = \Delta Q + \Delta W

    definition of the first thermodynamic law is very restrictive in that it is vague regarding details, and hence limiting.

    The usual treatment of a heat transfer problem is based on the approach that I started to describe. I mean that you allow for non-equilibrium in your system of interest, which necessarily introduces time as a parameter for the problem, a necessity if you want to say anything about fluxes.

    You then separate your system, the main control volume, into smaller subsystems, control volumes in themselves. This can be done, at least in principle, down to the scale where the hypothesis of local equilibrium holds. Averaging over such a subsystem must be stable enough over a typical timescale for the hypothesis to be sensible. This allows us to define the phenomenological (empirical black-box) fields, temperature, pressure and so on that is needed in thermodynamics, in a finite set of regions.

    Within one subregion, the temperature for instance is constant since a local equilibrium is established. Given that we treat a solid for instance, the local energy current over a control surface can easily be found from the temperature gradient over the surface using Fourier’s law. This is on the phenomenological level of course unidirectional since the real processes responsible for the energy transfer are averaged.

    Let us now think from a kinetic theory point of view. Let us consider a setting where we have two separate mono-atomic dilute gasses in two connected vessels, initially separated by a partition. These gases can be considered as ideal. Let one of the gases have a higher temperature. The energy in the system is only kinetic, stored in three quadratic degrees of freedom per atom as 1/2mv_i^2 for atom i. When releasing the partition the system will equilibrate. After doing so, we will find that

    1. The two vessels are at the same temperature.
    2. The two different types of atoms involved will be distributed over both vessels.

    Since the atoms don’t interact other than with vessel walls, there is no work involved.

    How can this be explained with a unidirectional transfer?

    Take this picture and move back to where we left our control surfaces. Do you see my point?

    • okulaer says:

      Voidness, you ask:

      “1. What do you mean by a field in this case? The electromagnetic field?”

      I feel I’m starting to repeat myself.

      The ‘bidirectional’ model of a radiative heat transfer specifically implies that two radiating objects facing each other both independently and oppositely transfer energy to the other one (no interaction, no mutual influence whatsoever). And, yes, these two simultaneous transfers simply go straight through each other inside the same space between the two objects. So the ‘field’ in this case can be interpreted both as simply the common space between the two objects and as the superimposed/integrated radiation field filling up this shared space between the two objects.

      That’s a ‘bidirectional transfer’. Which is fundamentally different from the cases you bring up.

      “2. How can this be explained with a unidirectional transfer?”

      So what you’re essentially claiming is that convective cells in fluids with density/temperature gradients are somehow examples of ‘bidirectional heat transfer’, that is, two opposing heat transfers moving directly through each other, occupying the exact same space without any interaction whatsoever, as if the opposing one weren’t even there, simply making up a ‘net’ transfer between them?

      Then how come ‘cells’ are formed at all? In a thunderstorm, is the cold air aloft flowing down straight through the warmer rising currents, thereby transferring ‘cold’ heat down to the surface? It is ultimately flowing down, after all. Something has to replace the air swiftly rising away from the surface.

      But are these ‘flows’ moving through each other? Or are they distinctly separated? And is there ever a heat transfer going on from cold tropospheric layers down to the warm surface? Or are these just ‘mass transfers’? You need to distinguish between the energy contained within a volume of air, its internal energy [U], and the transfers of energy to/from this same volume of air [W, Q] occurring as it comes into contact with other air masses.

      No, I’m afraid it doesn’t appear that you understand what I’m talking about when I distinguish between a ‘bidirectional’ and a ‘unidirectional’ heat transfer, Voidness. And frankly I find that a bit surprising, because it’s not a very hard distinction to follow … And you seem to be a clever guy.

      I don’t know what else to tell you …

      • Voidness says:

        “So what you’re essentially claiming is that convective cells in fluids…”

        No no, not at all. And I’m afraid that the point I’m trying to make seems to confuse you. The control volume is an abstraction that is necessary to build a continuum model (I’m trying to motivate how we can describe a system , that has on the order of (10^23)^3 (and up) degrees of freedoms with just a few parameters. I’m trying to move our attention to the microscopic processes (we are sub microns here, down to nanometers here) that actually do the transport of energy, and your attention goes to large macroscopic scales 🙂

        “But are these ‘flows’ moving through each other? Or are they distinctly separated?”

        I think it is necessary to look at the electromagnetic field here again. In vacuum the electromagnetic field must be a solution to Maxwell’s wave equation for the electric field (the magnetic field is then derived from Maxwell’s equations)

        d^2(E)/dt^2 = c^2 * ( d^2(E)/dx^2 + d^2(E)/dy^2 + d^2(E)/dz^2). (partial derivatives)

        The equation is linear, so if E_1 and E_2 are solutions, so are E = E_1 + E_2. This is a completely harmonic solution so E_1 and E_2 doesn’t interact in any way. If E_1 and E_2 “collide” they will only pass through each other, no influence what so ever.
        (there is speculations that there may be a non-linear correction to Maxwell’s wave equation in the case of very high energies, but that is not interesting in these low energy cases, and has never been experimentally confirmed…)

        This together with the earlier discussion we had, regarding coherence lengths and standing waves, would suggest that a bidirectional model isn’t just possible, it is also very plausible. The coherence length is as we already concluded on the order of 10^-5m scale. The optical path length is maybe from 1m and up, doesn’t really matter since this distance is arbitrary. If the optical path length is much greater then the coherence length, the relative phase between separate disturbances are random. The requirement for standing waves is stability in the phase difference.

        • okulaer says:

          “I’m trying to move our attention to the microscopic processes (we are sub microns here, down to nanometers here) that actually do the transport of energy”

          Are you indeed? Then I fear you will have to explain to me what precisely these ‘microscopic processes that actually do the transport of energy’ constitute. You’re setting up a situation where two gases at different temps at some point are allowed to mix. This will naturally happen by way of convection (advection+diffusion). Keep ‘mass transfers’ (advection) and ‘heat transfers’ (diffusion/conduction) apart, Voidness. Otherwise you’ll just confuse yourself into thinking you ‘see’ something you don’t. All the way down to an atomic/molecular level, the heat/energy transfer between the original warm subsystem and the original cool subsystem is invariably UNIdirectional, from warmer to cooler, despite the fact that ‘cool’ gas molecules of course will spread as much into the original ‘warm’ compartment as the ‘warm’ gas molecules spread into the original ‘cool’ compartment. There’s a mixing of the gases (via advection) and as a result, there’s an evening out of ‘internal energies’ [Us]. The Us even out by energy transferring from the (on average; again, I don’t care about individual cases) warmer gas molecules to the (on average) cooler gas molecules (via diffusion/conduction).

          “This together with the earlier discussion we had, regarding coherence lengths and standing waves, would suggest that a bidirectional model isn’t just possible, it is also very plausible. The coherence length is as we already concluded on the order of 10^-5m scale. The optical path length is maybe from 1m and up, doesn’t really matter since this distance is arbitrary. If the optical path length is much greater then the coherence length, the relative phase between separate disturbances are random. The requirement for standing waves is stability in the phase difference.”

          So we’re back at the specifics, are we? Individual cases. Back to the classical Newtonian view. A ‘bidirectional transfer’ model is not particularly plausible; not for its specifics, Voidness, but for the generality of the overall claim, that energy is freely transferred as ‘heat’ from cold to hot to make hot hotter. You still haven’t addressed at all that part of the ‘bidirectional’ explanation, that the cooler insulating layer alone is what ends up making the warmer heated object even warmer by transferring energy as ‘extra heat’ to it, the insulation ‘heating’ the insulated object. This is nonsense. And is the central issue as far as I’m concerned. This whole wave specifics thing is frankly not that interesting to me. Because, bottom line, it matters not. The EFFECT of radiative insulation is real. Whether or not this effect is explained UNIdirectionally (the insulating layer slows the cooling rate (reduces energy OUTPUT per unit time) of the heated object – plausible, physical) or BIdirectionally (the insulating layer enhances the heating rate (increases energy INPUT per unit time) of the heated object – implausible, un-physical), ultimately makes no difference. The atmosphere doesn’t radiatively warm the surface of the Earth anyway. It warms it by limiting its convective (and hence total) cooling rate at particular pressures and temperatures. This is a matter of MASS (heat capacity, weight), not of “back radiation” heating.

          (The claim ‘waves colliding don’t really interact at all’ is trivially and mathematically true. They simply superimpose. And travel through each other. Still, it is the superimposed form that is ‘real’ (as in ‘observed’). In a standing wave pattern, say, the opposing wave trains travel freely through each other. But we don’t see them doing so. We see only their superimposed wave pattern. The energy originally moving in each direction by the individual wave trains is now ‘caught’ in the resultant standing wave pattern.)

  15. Voidness says:

    Just a small addendum to “March 25, 2015 at 12:13 pm”

    The “wave equation” as it stands, are nonsense. I was in a bit of a hurry when writing…

    Please look at
    http://en.wikipedia.org/wiki/Electromagnetic_wave_equation
    instead.

    For some discussion on “colliding” waves, please see followin question at physics stackexchange

    http://physics.stackexchange.com/questions/114933/intuitive-explanation-of-the-waves-superposition

  16. Voidness says:

    “A ‘bidirectional transfer’ model is not particularly plausible; not for its specifics, Voidness, but for the generality of the overall claim, that energy is freely transferred as ‘heat’ from cold to hot to make hot hotter. You still haven’t addressed at all that part of the ‘bidirectional’ explanation, that the cooler insulating layer alone is what ends up making the warmer heated object even warmer by transferring energy as ‘extra heat’ to it, the insulation ‘heating’ the insulated object.”

    I’ve now managed to find a solution to all this.

    The insulating layer alone can never increase the temperature in the insulated object. That should be quite obvious. It took no more than 15 minutes to formulate and write a code for the heat transfer problem (should have done this way earlier… 🙂 ). The result showed what was expected, the insulating layer has the result of slowing down the cooling. You need to add an external source term in the model to get the temperature to rise in the insulated object. Seems very far fetched to say that the insulation “heats” the insulated object, other then in colloquial language.

    Then there is the issue with directionality.

    When I looked at the transient solutions to the model, it reminded me of something I have worked with as an engineer, a heat exchanger. Cold radiation gets equilibrated in the insulated body, and then resent as hotter radiation. So it is actually the insulated body that “heats” the radiation, and not the other way around. There is of course one big difference, the heat exchanger needs fans to drive the fluid to and from the exchanger. Here we get the transport for free since the radiation is massless and has zero chemical potential in the vacuum in between.

    Does this violate the second law? Set T_h > T_c ( => 1/T_h < 1/T_c). If energy of amount E is taken from the radiation field, entropy of the amount

    dS_cr = 4*E/(3*T_c)

    is taken from the radiation. In the insulated body entropy is produced, of the amount

    dS_hb = E/T_h.

    This gives

    dS_hb – dS_cr = E/T_h – 4*E/(3*T_c) 0,

    So the second law is ok.

    This also makes perfect sense with respect to the delay of the cooling. The insulated body needs to get rid of the extra energy, due to the second law, but this takes longer time since since the body can only dispose energy at a rate ~T_h^4 due to the Stefan-Boltzmann law.

    Another possibility for the body to dispose the energy at a faster pace is with help of an external power source that increases the temperature in the body and then also the rate of radiation.

    There really is no mystery here. With the picture of a heat exchanger in mind the problem gets turned on it’s head. The thermodynamic laws holds up, and on top of that gives a reasonable explanation for the observed behaviour.

    • Voidness says:

      There is a typo…

      In
      —————————————————–
      This gives

      dS_hb – dS_cr = E/T_h – 4*E/(3*T_c) 0,

      So the second law is ok.
      —————————————————–

      a ‘greater then’ sign is missing, it should read

      —————————————————–
      This gives

      dS_hb – dS_cr = E/T_h – 4*E/(3*T_c) > 0,

      So the second law is ok.
      —————————————————–

    • Voidness says:

      Never write in the middle of the night when you are tired… 🙂

      The check for the entropy production is royally … …

      The following should be ok.

      ————————————————————————————————————
      Does this violate the second law? Set T_h > T_c ( => 1/T_h < 1/T_c). If energy of amount E is taken from the radiation field, entropy of the amount

      dS_cr = 4*E/(3*T_c)

      is taken from the radiation. In the insulated body entropy is produced, of the amount

      dS_hb = E/T_h.

      This gives

      dS_hb – dS_cr = E/T_h – 4*E/(3*T_c) < E/T_h – 4*E/(3*T_h) = – E/(3*T_h) E/T_h + 4*E/(3*T_hr) – 4*E/(3*T_hr) = E/T_h > 0

      So the second law is ok.
      ————————————————————————————————————

    • okulaer says:

      Voidness,

      We appear simply to go in circles here. Our basic mindsets are clearly miles apart on this issue. You say:

      “The insulating layer alone can never increase the temperature in the insulated object. That should be quite obvious.”

      It is obvious. But it still manages to do so quite well, if you only follow the ‘bidirectional transfer’ model all the way through to its natural end.

      “The result showed what was expected, the insulating layer has the result of slowing down the cooling.”

      Sorry, Voidness, but I don’t know whether to laugh out loud or to heave a long-drawn sigh. This is EXACTLY what I’m talking about. We’re going around in circles. You don’t want to see the fundamental problem here. You want there to be a ‘bidirectional transfer’ in one.

      One more time: The problem is not in the effect. It is in the bidirectional explanation of the effect.

      Yes, the insulating layer slows the cooling of the insulated object. This is what we observe. But we cannot explain ‘slowed cooling’ of an object with a supply of MORE energy to it, by increasing its energy INPUT. Because that would be extra heating – the opposite process.

      We will have to explain it by what it really is – a process causing LESS energy escaping the object (per unit time), reducing its energy OUTPUT. That is what ‘slowed cooling’ is about.

      Take note: The energy output per unit time from the heated/insulated object is not reduced at any time in the ‘bidirectional transfer’ model. It is only increased. As a direct response to an increase in the energy input. (All of it from a cooler place.) And as a direct consequence of this extra input forcing its ‘internal energy’, and hence its absolute temperature, to rise. Enhanced cooling as a response to enhanced heating.

      We’re getting nowhere with this, Voidness.

      “You need to add an external source term in the model to get the temperature to rise in the insulated object. Seems very far fetched to say that the insulation “heats” the insulated object, other then in colloquial language.”

      Hehe, I see you’re turning the situation on its head. The warm object is first equilibrated with its heat/power source to reach a steady-state temperature. Then the cooler insulating layer is put up around the warm/heated object. And only then the temperature of the warm/heated object starts rising beyond its initial equilibrated temperature. But where is the extra energy effectuating this rise coming from? It’s not coming from the heat/power source. Nothing stops its energy from escaping the surface of the warm object. So there is no way for it to ‘pile up’ further after the initial equilibration. According to the principles of the ‘bidirectional transfer’ model.

      No, it comes solely from the cooler insulating layer now in place. Extra input. As if a second energy/heat/power source to the heated object were installed.

      That is extra ‘heating’, Voidness. Not ‘slowed cooling’. If anything, it is the colloquial term ‘slowed cooling’ that is far fetched. It is even downright deceiving, because it hides what’s actually going on in the ‘bidirectional transfer’ model: The cool object feeding the warm object with extra energy and thus heating it some more, its temperature rising as a direct response.

      Of course the insulating layer slows the cooling of the heated object, and of course that’s how its temperature rises further. It is what we observe.

      But what we do not observe is a ‘bidirectional transfer’ causing the temperature to rise further. We only hypothesize about it.

      However, from fundamental thermodynamic principles, such a process can only be properly explained by a ‘UNIdirectional transfer’ model. Because in such a model the warmer object actually cools (sheds its ‘internal energy’) more slowly. So it is the energy from its actual heat source that piles up inside it, making it warm. It doesn’t get heated (its ‘internal energy’ boosted) some extra by its cooler insulating layer (which is not a second heat source) to then cool (shedding its ‘internal energy) faster as a response.

      “There really is no mystery here.”

      We’ll try it one last time: The EFFECT does not violate the 2nd Law. The effect is real. Entropy is not reduced. It is only the ‘bidirectional transfer’ EXPLANATION of it that does. Because it distinctly describes the effect as extra heating of the warmer object by the cooler one, forcing the warmer object to warm even more in order to shed its energy faster.

  17. Voidness says:

    When thinking about it, all this confusion seems to originate from fuzzy definitions of heat and the second law of thermodynamics. No one is really to blame I think. The term heat is grossly used and misused when looking at applied sciences as within engineering for instance. I know that from a first hand experience (I have a masters degree in Engineering Physics). Within the physics community we have much parted from the word other than on an informal level, talking to each other. That is my feeling at least.

    I looked in the closest text book on statistical mechanics I have here, “Statistical Mechanics” by Pathria/Beale. It is a classic graduates text on statistical mechanics (who could tell 😀 ). It does not have one single reference to heat (other then heat capacity, which unfortunately is a bad misnomer that stuck for some reason). What you do find is proper definitions of entropy and thermodynamic potentials, as Helmholtz free energy, Gibbs free energy, enthalpy and so on. There is also no reference to the second law in any way, that is all contained in the concept of irreversibility.

    The trend continues when looking at some books treating applications, using thermodynamics lying arround. I have Ashcroft/Mermin’s “Solid State Physics”, Marder’s “Condensed Matter Physics”, Bergström/Goobar “Cosmology and Particle Astrophysics” here. Same thing, not one single reference to heat.

    The best thing would be to part from the word heat once and for all, it is a bad remnant from the completely crushed caloric theory, and has no real use since there are more fundamental quantities. It wouldn’t be missed…

    • okulaer says:

      Only you appear to be ‘confused’ by the concept of ‘heat’. Not me. Rather, the basic thermodynamic concept of ‘heat’ is so clear-cut it immediately ‘de-confuses’ the matter of ‘bidirectional’ vs. ‘unidirectional’ transfer.

      It is only when discarding the strict concept of ‘heat’ that confusion is allowed to arise when discussing/analyzing a ‘heat transfer’ process. Then all of a sudden ‘everything’ is game and free to happen. Hot heats cold, but cold also heats hot. What’s the problem? Just go with the flow.

      Your entire approach to this subject seems very much the result of just such confusion. You appear simply to not see (or wanting to see) the fundamental issue I’m pointing out.

      – – –

      ‘Heat’ is simply energy dynamically transferred (in transit) between regions or systems (as opposed to energy statically contained within those regions/systems) as a result of a temperature difference. It moves spontaneously only from higher to lower temperature. It is unidirectional. When two objects at different temps face each other, the warmer object spontaneously transfer some of its (internal) energy to the cooler object as ‘heat’. The cooler object does not simultaneously transfer any of its (internal) energy to the warmer object.

      The transfer of energy as ‘heat’ is simply what we always physically observe/detect in a ‘heat transfer’ process. It is a real, not a hypothetical/mathematically constructed, phenomenon like the UWLWIR or DWLWIR radiative ‘fluxes’ in Earth’s sfc-atm system.

      • Voidness says:

        I agree that there is nothing mysterious with heat as such, not when you use a proper definition and more importantly, not when we have equilibrium conditions.

        In Schroeders “An Introduction to Thermal Physics”, wich is a typical textbook given to undergradutes that takes an introductiory course on thermodynamics with statistical physics. It kind of nails down the problem we have here, regariding the nature of thermal radiation exchange.

        p. 37
        “Usually, to determine ‘what’ the equilibrium state of a system is, we need not worry about ‘how long’ the system takes to reach equilibrium states themselves. Thermodynamics, by many people’s defnitions, include only the study of equilibrium states themselves. Questions about time and rates of processes are then considered a separate (though related) subject, sometimes called transport theory or kinetics.
        In this book I won’t say much about rates of processes, because these kinds of questions are often quite difficult and require somewhat different tools.”
        p. 43
        “While kinetic theory is the most direct and cincrete approach to thermal pysics, it is also the most difficult. Fortunately, there are much easier methods for preceding most of the ‘equilibrium’ properties of materials, without having to know the details of how molecules move. To predict the ‘rates’ of processes, however, we usually have to resort to kinetic theory.”

        By construction, equilibrium thermodynamics (what you call thermodynamics here), can only treat situations where you start with one equlibrium and ends up with another equilibrium.

        I got really curious about this problem and made a quick literaure search. Some very interesting results popped up. The most important must be that the Stefan-Boltzmann law by no means are universal. There are several experiments showing a violation of the originally stated Stefan-Boltzmann law, in the case of close proximity (within micrometers). This is probably due to near-field effects (tunneling of evanescent wave).

        Fortunately, since I don’t really have the time to do so myself, I also found a non-equilibrium treatment of the problem, leading up to a genrealized Stefan-Boltzmann law, published here

        http://www.degruyter.com/view/j/jnet.2010.35.issue-3/jnetdy.2010.017/jnetdy.2010.017.xml

        ( You can get a copy here http://www.ffn.ub.es/webmrubi/papers/127_mrubi.pdf )

        This paper is a formidable avalanche over the proposition that there should be only a one way transfer. This issue should now be settled.

        • okulaer says:

          Thanks 🙂 I’ll read it when I find the time …

        • okulaer says:

          Voidness,

          The people behind the paper you linked to start by saying: “We show that the system evolves to a stationary state characterized by an energy current which satisfies a law similiar to the Stefan–Boltzmann law. The magnitude of this current depends on the temperature of the emitters expressed through the difference of the fourth power of these temperatures.” (My emphasis.)

          The ‘energy current’ in question is the TRANSFER OF ENERGY that I’ve been talking about all this time. It is invariably UNIDIRECTIONAL, spontaneously moving from hot to cold. Their ‘net current of heat’. It is ALL that is ever actually detected/observed in a heat transfer like this. In the real world. We know only of this UNIdirectional transfer of energy. Everything else, everything beyond this, is pure theory. The authors of your paper do exactly what all physicists appear to be doing – uncritically adhering, by spinal reflex, to the conceptual ‘bidirectional flow’ explanation of radiant heat transfer, starting out by automatically assuming a two-way transfer making up a ‘net’:

          “(…) to undertake this study, we assume that the dynamics of the photons is the result of two simultaneous processes: emission and absorption of cold photons at T_C and emission and absorption of hot photons at T_H; this is illustrated by the Figure 1.” (My emphasis.)

          This is simply the commonly imagined model, Voidness. What’s new?

          The crucial point here is that the authors of this paper have no right to define a state characterized by a ‘(net) energy current’ [Q] at all. Not if they want to stick to and go by the ‘bidirectional flow’ model. There is no ‘net current’ to be generated by two separate, opposing currents that do not interact in the least within the space where this ‘net current’ is supposed to spontaneously appear. If my water gun blasts you with a powerful jet of water and you blast me back with a jet of only half the pressure, we do not generate between us a ‘net’ stream of water of [1-0.5=] 0.5p moving towards you only, leaving you half as wet and me completely dry. We will both be soaked.

          There is no one ‘heat current’ in a bidirectional transfer. There are TWO ‘heat currents’ working in complete isolation from one another, as if the opposite one weren’t there, still occupying the exact same space, the ‘hot current’ heating at the cold end AND the ‘cold current’ heating at the hot end.

          The ‘heat’ [Q] in this situation is really the ‘net heat’ and will not in itself be a vector or a current moving through the space between the objects involved. It will not itself be a ‘transfer of energy’. It will be nothing physical. It will only be a number. A quantification, the arithmetical net RESULT at each end of the bidirectional transfer of the two separate emissions and absorptions (alternatively, ‘momenta’) of the opposing photon heat currents.

  18. Voidness says:

    This blog doesn’t seem to process equations in the comments that well, or maybe it’s just me 😉

    Comment:

    ‘To heat a planetary surface’ for dummies; Part 4

    (March 27, 2015 at 1:29 am)

    is of course so important that it really needs correct typesetting for the math. I rewrote it in LaTex instead and give it as a pdf as following link:

    https://www.dropbox.com/l/xm1PGYzkkm7ls0NH9uFKmo

  19. SSv says:

    Hi!

    I apologize if this has been asked already. I have not read every comment here.

    Say you have two isolated interacting blackbodies at different temperatures.

    If there is bidirectional transfer, do you mean that this must result in the hotter body getting an even higher temperature and the colder an even lower temperature?

    • okulaer says:

      Uhm, no. I mean the opposite.

      • SSv says:

        Then I don’t understand how that interaction would violate the second law of thermodynamics (2nd LOT).

        The 2nd LOT is a statement of direction of physical processes.

        One way Clausius expressed this was by saying that in nature you never see a process were a hotter body gets an increased temperature and a colder body a lower temperature when they are in thermal contact, given positive heat capacities. That is what heat from cold to hot means.

        So the statement that you make, that bidirectional energy transfer would violate the 2nd LOT looks very strange. Especially given your clarification here.

  20. SSv says:

    You seem to agree that there is no thermodynamic obstacle to bidirectional energy transfer. Next step is to recognize that the coherence length is on the 1-10 micron scale and that a typical separation reasonably is much greater than the coherence length. The problem can then be treated purely ballistic as rays transporting the energy and there is no interference between the two sources. So the transfer really is in both directions.

    • SSv says:

      Just a clarification and summary.

      Since the blog post considers the Stephan-Boltzmanns law blackbodies are the considered type of systems. Since blackbodies per definition absorbs and thermalizes all incident radiation the phase-information is completely lost. This means that the radiation from one body is completely independent of the other and vise versa as long as the separation distance is longer than the coherence length. That is typically around 10 micrometers at room temperature and smaller for higher temperatures.

      This description, using a model of ballistic transport and ray-optics is the standard one in an ordinary engineering description of radiative heat transfer. This has been tested both in theory and practice countless times.

      To summarize,

      1. There are no thermodynamic limitations to the existence of a two way transfer of internal energy between two blackbodies.

      2. The combined system is preferably described by a ballistic approach since there is no coherent phases between the two different systems. This has the implication that energy is transfered by a process that preferably is described with simple ray-optics between the two interacting bodies and has the meaning that internal energy is transfered in both directions.

      3. The body with higher temperature radiates more per unit time so the heat transfer is always from the body with higher temperature to the body with lower temperature.

    • okulaer says:

      SSv, we have already discussed this subject at length on this very thread. Voidness and me. We never reached an agreement. But this issue is really not critical at all to my overall argument. It’s an aside at best. We cannot physically observe bidirectional flow, only ever unidirectional radiant heat, and therefore it all comes down to theoretical concepts and how you wish to describe it. No one can know.

      However, if you’d read my posts on this, you would see that I am not arguing that the idea of ‘radiative insulation’ in itself violates 2LOT. Not in the least. I am arguing that using the bidirectional principle to EXPLAIN it ends up violating it, not, however, when viewing the interaction between two cooling bodies alone, but when following the separate energy flows within a three-body system, that is, if the one body is already constantly being heated by a third body (or ‘heat source’), meaning its temperature is not allowed to drop. This is the situation described by “climate science”, and that’s where the flux down from the atmosphere ends up being the energy flow directly raising the temperature of the surface in absolute terms. In going from 232 to 289K, the surface specifically needs the extra input of energy from the cooler atmosphere. Which is highly problematic in terms of EXPLAINING the higher temperature from insulation.

      Thanks for posting 🙂

      • SSv says:

        We seems to agree to the fact that there is nothing inherently wrong with a simultaneous energy transfer in both directions. Even in the case we’ve been discussing, that is the closed interaction of two bodies.

        There is really no difference in the three-body case, the distribution of energy is of course a bit different than in the case of two interacting systems. The case that you then introduce is a completely different one. When you go to the case of the Earth the system can no longer be considered closed and in equilibrium. You have one source in the form of incoming radiation. And one sink in the form the empty space. That strongly affects the applicability of the 2nd LOT in relation to the earth system. We are no longer talking about the transition from one equilibrium state to another that would be the case when two isolated bodies interact, as we have discussed here above. On top of that there is not always an equilibrium or a non-equilibrium close enough to a equilibrium so that thermodynamic quantities are well defined. So the laws of thermodynamics are not really valid in a technical sense.

        Kinetic theory of course seems more reasonable, right?

        How the energy ends up in the sink is a very complicated process involving the ordinary modes of energy transfer, conduction, convection, radiation and mass transfer. Radiation is the mode of transfer from the Earth system into the sink, both directly from the surface and from the atmosphere that mainly gets it’s energy through convective loss from the surface.

        A very important factor to consider then is if radiative transport from lower layers to the sink is ballistic of diffusive. The fastest transport is of course a ballistic one. In the quasistatic case this would result in a certain amount of energy within the earth system corresponding to an average temperature of X Kelvin measured over a representative set of local equilibriums. Any kind of scattering mechanisms added to this system would result in a diffusive transport where energy from Earth dissipates to the sink at a slower rate than in the ballistic case. This of course means a larger amount of energy in the Earth system and that would in the quasistatic case result in an even higher average temperature (ignoring the cases of phase transitions) of Y Kelvin where Y > X.

        Are you familiar with microscopic theory of thermal conductivity in solid states? The idea above isn’t really more “magical” than thermal conductivity in a solid. The vibrations are scattered by different mechanisms. Stronger scattering means less conductivity and slower dissipation of energy.

  21. climategrog says:

    Hi, I just saw your post on WUWT , rather belatedly , today.

    Climate change debate – latest results

    those graphs, especially the last one, are very interesting. They support what I found when looking at tropical feedbacks: that there is and extra net heat input to the system following Mt Pinatubo eruption.

    TLS suggests a similar effect after El Chichon.

    On Determination of Tropical Feedbacks

    ie there is a WARMING effect from major stratospheric eruptions which is not accounted for in climate modelling and it being falsely attributed to AGW

    There was also a clear cooling trend which started a good year before Mt P which usually get confounded with the true volcanic effect leading to excessive estimations of the volcanic cooling which are then balanced by excessive estimations of GHE warming.

Leave a comment